Sunteți pe pagina 1din 139

NDICE

Pgina

Contenido
Unidad 1
Parfrasis...........................................................................................

Tecnicismos......................................................................................

14

Locuciones latinas............................................................................
Abreviaturas bibliogrficas..................................................................

20
24

Anlisis crtico de textos expositivos.............................................


Pasos para realizar un anlisis crtico de textos u obras expositivas
Aspectos a analizar en una obra expositiva........................................
Clasificacin de los obras o textos expositivos...................................
Obras cientficas.................................................................................
Obras didcticas.................................................................................
Obras de divulgacin..........................................................................

26
26
28
29
30
32
34

Anlisis de textos informativos.......................................................


El peridico.........................................................................................
La noticia.............................................................................................
Gneros periodsticos.........................................................................

36
36
39
41

Anlisis de textos literarios.............................................................


Obras narrativas................................................ ................................
El cuento................................................ ...........................................
La novela................................................ ...........................................
Obras dramticas................................................ ...............................
La tragedia................................................ .........................................
La comedia................................................ .........................................
Obras lricas................................................ .......................................
El himno................................................ .............................................
La oda................................................ ................................................
La elega................................................ ............................................
La cancin................................................ ..........................................

43
43
44
47
49
50
51
53
53
54
55
55

Unidad 2
Trabajo Acadmico...........................................................................
Tipos de Investigacin................................................ .......................
Estructura del trabajo acadmico........................................................
Presentacin del trabajo acadmico...................................................

60
61
65
77

Informe (Clasificacin y Estructura)..................................................

79

Ensayo...............................................................................................

83

Monografa.........................................................................................

87

Autoevaluacin. 90

Unidad 3
Expresin Oral....................................................................................
Principios bsicos para hablar eficazmente: Adquisicin de los
conocimientos, Autodominio y Canalizar el nerviosismo....................
Malos hbitos que todo orador debe evitar.........................................
Buenos hbitos que todo orador debe desarrollar..............................

97

Debate.................................................................................................
Funcin del debate..............................................................................
Debate acadmico..............................................................................

109
110
111

Disertacin.........................................................................................

115

Discurso..............................................................................................
Tipos de discurso...............................................................................
Caractersticas del buen conferenciante.............................................
Organizacin de un tema y su presentacin a un auditorio.................

117
117
121
123

98
105
105

Autoevaluacin.. 132
Bibliografa........................................................................................... 138

U
N
I
D
A
D

Lectura
3

PARFRASIS

La parfrasis es un trmino de origen griego que se compone de dos elementos:


Par: que significa junto a, al lado de.
Frasis: que significa decir.

Parfrasis:
Es la explicacin o interpretacin
ampliada que se hace de un texto,
con el propsito de que ste sea
ms claro y comprensible.

Hacer la parfrasis de un texto, significa utilizar nuestras propias palabras para redactar su
contenido, considerando los aspectos de mayor significado, sin alterar la idea original del
autor. As, es posible parafrasear textos informativos, tcnicos, expresivos o literarios. Por
consiguiente, parafrasear es una habilidad que requiere de gran capacidad de
comprensin como lector, puesto que, comprender el mensaje de un texto, equivale a
realizar la mitad del proceso comunicativo, pues la intencin de quien escribe es provocar
una reaccin de quien recibe el mensaje.

Cuando leas un texto y seas capaz de explicarlo con tu propio vocabulario, habrs
comprendido la lectura y tendrs la capacidad de discernir sobre cualquier tema.

Pasos para realizar una parfrasis:


1.- Lee cuidadosamente el texto.
2.- Consulta en el diccionario las palabras que no hayas entendido.
3.- Identifica la idea principal.
4.- En caso de ser necesario, sustituye las palabras clave por un sinnimo.
5.- Redacta el borrador de la parfrasis.
6.- Realiza la redaccin definitiva.

Ventajas que ofrece la parfrasis como forma de expresin:


Ayuda a explicar y comprender mejor el mensaje de un texto.
Contribuye a aprender ms adecuadamente la informacin que proporciona un
texto.
Manifiesta la forma de respuesta por parte del lector.
Conduce a un mejor desarrollo de las capacidades de estudio.
Constituye un recurso til para la elaboracin de fichas de trabajo.
Apoya a la preparacin de apuntes para examen o exposicin de clase.
Propicia la seleccin de un vocabulario adecuado.

EJEMPLO:
TEXTO ORIGINAL
DEMOGRAFA
La demografa (del griego demos, pueblo y grapho, describir), es una
importante disciplina auxiliar de la Economa que tiene por base la estadstica de la
poblacin. Si para la poltica econmica es imprescindible el conocimiento de los
recursos naturales y de los bienes de capital disponibles, es an ms interesante
saber con qu potencial humano se cuenta y cmo est constituido. En Mxico, los
Censos Generales de Poblacin se levantan cada diez aos y captan diversos
informes sobre las caractersticas de los individuos, como son: sexo, edad, grado
de escolaridad, alimentacin, ocupacin, estado civil, etc. Esto permite adems,
conocer los diferentes niveles socioeconmicos de la poblacin y la posibilidad de
desarrollo y progreso con que cuentan. Se han realizado hasta la fecha 12 censos
de este tipo. La informacin censal se presenta en un resumen general, con datos
generales y por entidades federativas. Adems, se publica en volmenes
especiales para cada uno de los estados de la Repblica, con cifras por
municipios. La elaboracin de estas estadsticas permanentes, permite utilizar los
datos antes mencionados sobre la poblacin del pas.

PARFRASIS
DEMOGRAFA
La palabra demografa, proviene del griego demos, que significa pueblo y grapho,
describir.
La demografa pertenece a la Economa ya que permite describir de manera
detallada todas las actividades que una poblacin realiza. As como conocer el
potencial humano con el que cuenta y cmo est constituido.
En Mxico, a travs de los Censos Generales, es como se recaba la informacin
para dar a conocer cuntos somos, a qu nos dedicamos, cmo vivimos, si
estudiamos o trabajamos, si somos solteros o casados, etc. Permitiendo as
conocer los diferentes niveles socioeconmicos que existen en nuestro pas. Todo
esto para visualizar a futuro las oportunidades de desarrollo y progreso con las que
se puede contar.
Es importante mencionar que toda esta informacin se hace llegar en volmenes a
cada estado y municipio que integran la Repblica Mexicana.
Es as como los Censos Generales son un gran apoyo para saber cmo est
constituida la demografa en determinado sector o pas.

Actividad 1: Lee cuidadosamente el siguiente texto y subraya las palabras cuyo


significado desconozcas.

CIEN AOS DE SOLEDAD


(FRAGMENTO)
Gabriel Garca Mrquez.
De acuerdo con los clculos de Jos Arcadio Buenda, la nica posibilidad de contacto
con la civilizacin era la ruta del norte. De modo que dot de herramientas de desmonte y
armas de cacera a los mismos hombres que lo acompaaron en la fundacin de
Maconda; ech en una mochila sus instrumentos de orientacin y mapas, y emprendi la
temeraria aventura.
Los primeros das no encontraron un obstculo apreciable. Descendieron por la
pedregosa ribera del ro hasta el lugar donde aos antes haban encontrado la armadura
del guerrero, y all penetraron al bosque por un sendero de naranjos silvestres. Al trmino
de la primera semana, mataron y asaron, pero se conformaron con comer la mitad y salar
el resto para los prximos das. Trataban de aplazar con esa precaucin la necesidad de
seguir comiendo guacamayas cuya carne azul tena un spero sabor de almizcle. Luego,
durante ms de diez das, no volvieron a ver el sol. El suelo se volvi blando y hmedo,
como ceniza volcnica, y la vegetacin fue cada vez ms insidiosa y se hicieron cada vez
ms lejanos los gritos de los pjaros y la bullaranga de los monos, y el mundo se volvi
triste para siempre.

Actividad 2: Realiza las actividades correspondientes.


a) En el siguiente espacio, anota las palabras que subrayaste, consltalas en el
diccionario y escribe su significado en las lneas.
_________________________________________________________________________________
_________________________________________________________________________________
_________________________________________________________________________________
_________________________________________________________________________________
_________________________________________________________________________________
_________________________________________________________________________________
_________________________________________________________________________________
b) Escribe los sinnimos que creas que expliquen mejor el texto.
7

hombres

por

individuos

_________________________ por _________________________


_________________________ por _________________________
_________________________ por

_________________________

_________________________ por

_________________________

c) Realiza la redaccin de la parfrasis del texto anterior.


________________________________________________________________________
________________________________________________________________________
________________________________________________________________________
________________________________________________________________________
________________________________________________________________________
________________________________________________________________________
________________________________________________________________________
________________________________________________________________________
________________________________________________________________________
________________________________________________________________________
________________________________________________________________________
________________________________________________________________________
________________________________________________________________________
________________________________________________________________________
________________________________________________________________________
________________________________________________________________________
________________
________________________________________________________________________
_
________________________________________________________________________
________________________________________________________________________
__
________________________________________________________________________
_

________________________________________________________________________
_
________________________________________________________________________
_
________________________________________________________________________
_

Actividad 3: Lee los siguientes textos y realiza las actividades correspondientes.


AMOR TARDO
Annimo
Nunca pens en el martirio
de un amor que nace tarde,
ni pude sospechar nunca
que as me martirizase.
Por eso es ms la tristeza
que hoy a mi corazn invade
al ver que tengo que huir
del riesgo como un cobarde.
Crceme yo en tu camino
o t el mo atravesaste , que
todava no s quin de los
dos fue el culpable.
Y a pesar de las distancias
que apartan nuestras edades,
tu alma y la ma se buscan
en miradas calcinantes,
sufriendo la sed terrible
de una pasin insaciable
que nos marchita, que nos
abate.
Porque esto es una locura.
No lo oyes, Mara del
Carmen?
Una insensatez, un sueo,

un delirio irrealizable.
No ves que nunca las
canas fueron galas de
galanes?
Que en ti alumbra la aurora,
y en m declina la tarde, y
que el viento del otoo deja
desnudos los rboles,
arrancando de sus ramas
ilusiones e ideales.
Mi experiencia es de un
demonio, y tu candor de un
ngel; y no es bien que
vayan juntas tu bondad y
mis maldades.
Conque, retira ese fuego,
o haz que la llama se
apague, que los crteres de
nieve no riman con los
volcanes, ni es natural que
una rosa cambie un clavel
por un sauce...
Vamos, bien mo! S
razonable, suelta mi mano,

a) Escribe la idea central del texto.


9

sigue adelante, djame


octubre con sus recuerdos,
pdele a mayo sus realidades:
la primavera llena de flores
est llamndote. Vete, s,
que a m me falta valor y
fuerza para dejarte; porque
sabes que te quiero como no
he querido a nadie, que es
invencible la fuerza con que
tus ojos me atraen; que si
renuncio, es a costa de una
voluntad gigante: que el verte
es morir de angustia, que el
no verte es sepultarme; que el
huir es vergonzoso y el
sucumbir es infame; que eres
mi cumbre, mi luz, mi gua,
y eres mi abismo, mi horror,
mi crcel.
Oh, mujer que en mi camino
te encuentro para mis males!
Por qu he nacido tan
pronto!
Por qu naciste tan tarde!

________________________________________________________________________
________________________________________________________________________
________________________________________________________________________
___

b) Realiza la parfrasis del texto titulado: Amor Tardo.


________________________________________________________________________
________________________________________________________________________
________________________________________________________________________
________________________________________________________________________
________________________________________________________________________
________________________________________________________________________
________________________________________________________________________
________________________________________________________________________
________________________________________________________________________
_________
c) Lee los siguientes refranes y parafrasalos en el espacio correspondiente.
1.- No dejes para maana lo que puedas hacer hoy.
________________________________________________________________________
________________________________________________________________________
__
________________________________________________________________________
_
2.- Andamos arando, dijo la mosca al buey.
________________________________________________________________________
________________________________________________________________________
__
________________________________________________________________________
_
10

3.- Al mal tiempo, buena cara.


________________________________________________________________________
_
________________________________________________________________________
________________________________________________________________________
__
4.- A palabras necias, odos sordos.
________________________________________________________________________
________________________________________________________________________
________________________________________________________________________
___

5.- Con la vara que midas sers medido.


________________________________________________________________________
________________________________________________________________________
__
________________________________________________________________________
_
d) Define los siguientes vocablos con tus propias palabras.
1.- Compaero: ___________________________________________________________
2.- Copiar:

___________________________________________________________

3.- Amor:

___________________________________________________________

4.- Amistad:

___________________________________________________________

5.- Escuela:

___________________________________________________________

6.- Honradez:

___________________________________________________________

7.- Libertad:

___________________________________________________________

8.- Justicia:

___________________________________________________________

9.- Solidaridad: ___________________________________________________________


10.- Estudiante: ___________________________________________________________
11

11.- Vacaciones: ___________________________________________________________


12.- Exmenes: ___________________________________________________________

El Gusano y el Escarabajo
Haba una vez un gusano y un escarabajo que eran amigos, pasaban charlando horas y
horas. El escarabajo estaba consciente de que su amigo era muy limitado en movilidad,
tena una visibilidad muy restringida y era muy tranquilo comparado con los de su
especie. El gusano estaba muy consciente de que su amigo vena de otro ambiente,
coma cosas que le parecan desagradables y era muy acelerado para su estndar de
vida, tena una imagen grotesca y hablaba con mucha rapidez. Un da, la compaera
del escarabajo le cuestion la amistad hacia el gusano. Cmo era posible que
caminara tanto para ir al encuentro del gusano? A lo que l respondi que el gusano
estaba limitado en sus movimientos. Por qu segua siendo amigo de un insecto que
no le regresaba los saludos efusivos que el escarabajo haca desde lejos? Esto era
entendido por l, ya que saba de su limitada visin, muchas veces ni siquiera saba
que alguien lo saludaba y cuando se daba cuenta, no distingua si se trataba de l para
contestar el saludo, sin embargo call para no discutir. Fueron muchas las respuestas
que en el escarabajo buscaron para cuestionar la amistad con el gusano, que al final,
ste decidi poner a prueba la amistad alejndose un tiempo para esperar que el
gusano lo buscara.
Pas el tiempo y la noticia lleg: el gusano estaba muriendo, pues su organismo lo
traicionaba por tanto esfuerzo, cada da emprenda el camino para llegar hasta su
amigo y la noche lo obligaba a retornar hasta su lugar de origen. El escarabajo decidi
ir a ver sin preguntar a su compaera qu opinaba. En el camino varios insectos le
contaron las peripecias del gusano por saber qu le haba pasado a su amigo. Le
contaron de cmo se expona da a da para ir a dnde l se encontraba, pasando cerca
del nido de los pjaros. De cmo sobrevivi al ataque de las hormigas y as
sucesivamente. Lleg el escarabajo hasta el rbol en que yaca el gusano esperando
pasar a mejor vida. Al verlo acercarse, con las ltimas fuerzas que la vida da, le dijo
cunto le alegraba que se encontrara bien. Sonri por ltima vez y se despidi de su
amigo sabiendo que nada malo le haba pasado. El escarabajo avergonzado de s
mismo, por haber confiado su amistad en otros odos que no eran los suyos, haba
perdido muchas horas de regocijo que las plticas con su amigo le proporcionaban. Al
final entendi que el gusano, siendo tan diferente, tan limitado y tan distinto de lo que l
era, era su amigo, a quien respetaba y quera no tanto por la especie a la que
perteneca sino porque le ofreci su amistad.
El escarabajo aprendi varias lecciones ese da: La amistad est en t y no en los
dems, si la cultivas en tu propio ser, encontrars el gozo del amigo. Tambin entendi
12

que el tiempo no delimita las amistades, tampoco las razas o las limitantes propias ni
las ajenas. Lo que ms le impact fue que el tiempo y la distancia no destruyen una
amistad, son las dudas y nuestros temores los que ms nos afectan. Y cuando pierdes
un amigo una parte de t se va con l. Las frases, los gestos, los temores, las alegras e
ilusiones compartidas en el capullo de la confianza se van con l.
El escarabajo muri despus de un tiempo. Nunca se le escuch quejarse de quien mal
le aconsej, pues fue decisin propia el poner en manos extraas su amistad, slo para
verla escurrirse como agua entre los dedos.
Si tienes un amigo no pongas en tela de duda lo que es, pues sembrando dudas
cosechars temores. No te fijes demasiado en cmo habla, cunto tiene, qu come o
qu hace, pues estars poniendo en una vasija rota tu confianza. Reconoce la riqueza
de quien es diferente de t y est dispuesto a compartir sus ideales y temores, pues
esto alimenta el espritu de supervivencia ms que un buen platillo. La esencia del
gusano y el escarabajo se volvi una en el plano que se encuentra ms all de este
mundo, volviendo al regocijo que en esta vida haban encontrado.
Este es el final de la historia, pues siendo t mi amigo no te puedo exponer a una
tristeza que no quisiera para m. No s si seas el gusano o yo el escarabajo, pero
seguro que somos distintos y en planos ajenos nos movemos. Yo, como gusano, te
seguir buscando da a da, y como escarabajo, no me fijar en limitaciones. Como
gusano, omitir lo grotesco que me puedas parecer. Como escarabajo, har uso de mis
habilidades para servirte.
Realiza una parfrasis del texto El Gusano y el Escarabajo.
________________________________________________________________________
________________________________________________________________________
________________________________________________________________________
________________________________________________________________________
________________________________________________________________________
________________________________________________________________________
________________________________________________________________________
_______
________________________________________________________________________
________________________________________________________________________
________________________________________________________________________
________________________________________________________________________
________________________________________________________________________
________________________________________________________________________
________________________________________________________________________
13

________________________________________________________________________
________________________________________________________________________
_________
________________________________________________________________________
________________________________________________________________________
________________________________________________________________________
________________________________________________________________________
________________________________________________________________________
________________________________________________________________________
________________________________________________________________________
_______
________________________________________________________________________
________________________________________________________________________
________________________________________________________________________
________________________________________________________________________
____
________________________________________________________________________
________________________________________________________________________
__
________________________________________________________________________
_
________________________________________________________________________
_

TECNICISMOS

Existen muchas palabras en castellano que se han formado de la unin de los


gramemas semiautnomos, voces griegas o latinas que en su idioma del origen son
autnomos, pero no en espaol. Es por esto, que existen palabras en nuestra lengua que
14

se forman anteponiendo y posponiendo a la palabra primitiva un prefijo o un sufijo


respectivamente.
Pues bien, las palabras as formadas son las que generalmente reciben el
nombre de tecnicismos, por que son propias de una tcnica, ciencia o profesin.

Tecnicismos:
Son palabras formadas por prefijos o
sufijos de origen griego o latino, o por
dos o ms voces del mismo origen. Los
tecnicismos son propios de un lenguaje
especializado.

As la tecnologa, las ciencias y las diversas profesiones tienen sus propios


tecnicismos para definir o catalogar a las personas, las cosas, acciones, etc.
Para reafirmar lo anterior, presentamos algunas palabras tcnicas (tecnicismos)
de uso comn en diversas especialidades.

A.
Tecnicismo
Topografa

EN INGENIERA
Origen
Topos = lugar

Significado
Descripcin de la superficie o de un terreno.

Graf = descripcin
Hipogeo

Hipo = debajo

Que crece, vive o se desarrolla bajo la tierra.

Geo = tierra
Aritmgrafo

Aritmos = nmero

Instrumento para realizar mecnicamente las

Graf = descripcin

operaciones matemticas.
15

Agrologa

Agrs = campo

Estudio del suelo en relacin con la vegetacin.

Lgos = estudio

B. EN FILOSOFA
Tecnicismo
Hedonismo

Empirismo

Axiologa

Origen
Edon = placer

Significado
Sistema moral en el que el placer es el

Ismo = sistema

fin supremo de la vida.

Empeira = experiencia

Sistema o procedimiento basado en la

Ismo = sistema

experiencia.

Axios = valor

Estudio filosfico de los valores.

Lgos = estudio o tratado


Autocracia

Autos = mismo

Sistema de gobierno basado en la

Cratos = poder

voluntad de un solo hombre.

C. EN MEDICINA
Tecnicismo
Amigdalitis

Origen
Amigdale = amgdala

Significado
Inflamacin de las amgdalas.

Its = inflamacin
Hepatitis

par y ats = hgado

Inflamacin del hgado.

Ita = inflamacin
Estetoscopio

Stetos = pecho

Aparato para examinar el pecho.

Scopa = ver, examinar


Carcinoma

Carcnos = cncer

Tumor canceroso.

Ma = efecto o resultado de la
accin

16

D. EN MATEMTICAS
Tecnicismo
Geometra

Origen
Geos = tierra

Significado
Estudia las propiedades y

Ecuacin

Metrn = medida
Aecuare = igualar

medidas de la tierra.
Igualdad que contiene una o

Tringulo

Tra = tres

ms incgnitas.
Figura de tres ngulos.

ngulus = ngulo

TECNICISMOS EN ESPAOL (VOCES GRIEGAS O LATINAS)

Voz griega o latina


Aeri, aero
Agros
Algis
Antropos
Astro, aster
Auto
Bi, biz, bis
Bios
Cosmos
Cronos
Metro
Mono
Morfo
Multi
Noma
Poli
Deca
Filos
Fona, fnica

Significado
Aire
Campo
Dolor
Hombre
Estrella
Uno mismo
Dos veces
Vida
Mundo, universo
Tiempo
Medida
Uno
Forma
Mucho
Gobierno, ley
Mucho
Diez
Amor, amigo
Sonido
17

Ejemplo
Aeropuerto
Agrnomo
Neuralgia
Antroplogo
Astrologa
Autodominio
Bicolor
Biologa
Cosmopolita
Cronmetro
Termmetro
Monoslaba
Morfologa
Multifactico
Autonoma
Polisemia
Dcada
Filosofa
Afnico

Gastri
Grafo
Gono
Hidro
Pseudo
Tcnia, tcnica
Tele
Termo
Voro

Estmago
Escritura
ngulo
Agua
Falso
Arte, oficio
Distancia
Caliente
Que devora

Gastritis
Ortografa
Polgono
Deshidratar
Pseudnimo
Tecnologa
Televisin
Termmetro
Carnvoro

Actividad 4: Realiza los siguientes ejercicios.

a)
C
onsulta en el diccionario el significado de los siguientes tecnicismos y escrbelos en las
lneas.
Afona: __________________________________________________________________
Politcnico: _______________________________________________________________
Filntropo: ________________________________________________________________
Multiforme: _______________________________________________________________
Omnvoro: ________________________________________________________________
Pirotecnia: ________________________________________________________________
Telfono: _________________________________________________________________
Polglota: _________________________________________________________________
Zoologa: _________________________________________________________________
Autgrafo: ________________________________________________________________
Trigonometra: ____________________________________________________________
Polinomio: ________________________________________________________________
Dermatitis: _______________________________________________________________
Pericardio: _______________________________________________________________
Petrografa: _______________________________________________________________
Ortografa: _______________________________________________________________
Acfalo: __________________________________________________________________
18

Arteriosclerosis: ___________________________________________________________

b) Relaciona cada vocablo con su definicin etimolgica. Escribe en la lnea del centro el
nmero que corresponda.
1) Politcnico
2) Bgamo
3) Antroplogo
4) Cosmopolita
5) Eufona
6) Amorfo
7) Cefalalgia
8) Astronoma
9) Zoologa
10) Geografa

______
______
______
______
______
______
______
______
______
______

Casado dos veces


Estudia a los hombres
Ciudadano del mundo
Muchas tcnicas
Reglas que rigen los astros
Sonido agradable
Sin forma
Estudio de los animales
Descripcin de la tierra
Dolor de cabeza

c) Escribe dos tecnicismos para cada una de las siguientes voces griegas o latinas.
LOGA....................................Tratado o estudio
Lexicologa.......................... Tratado de las palabras
______________________:_____________________________________________
______________________:_____________________________________________
METRA.................................Medida
Cronmetro.......................... Medida del tiempo
______________________:_____________________________________________
______________________:_____________________________________________
NOMIN................................. Nombre
Annimo............................. Sin nombre
______________________:_____________________________________________
______________________:_____________________________________________
CULTURA..............................Cultivo
Agricultura........................... Cultivo de los campos
______________________:_____________________________________________
______________________:_____________________________________________
19

LOCUCIONES
LATINAS

En nuestro idioma, en el habla comn se utilizan algunas palabras o frases latinas.


Estas expresiones se llaman locuciones latinas.
Ejemplo de
fragmento:

las locuciones latinas (en negritas y cursivas) usadas en el siguiente

Eran las 8:00 a.m. (antes meridiano) cuando Jorge Gonzlez, alias El Galn, sala
an somnoliento de su negocio, en donde haba estado trabajando. Se notaba
preocupado pues el negocio de compra-venta de muebles, que era su modus vivendi,
no marchaba bien. Haba hecho un balance de sus finanzas y stas arrojaban un
dficit considerable. Se encontraba a punto de la quiebra.
Una hora ms tarde lleg a las puertas de aquel alern que, ad hoc, haba mandado
aos atrs construir y en donde fabricaba y reparaba los muebles que le requeran.
Abri la puerta, entr y se sent, pensativo, tras su pequeo escritorio, sobre el que
encontr un telegrama en el que le comunicaban que su to abuelo, Eduardo, que viva
en otra ciudad, haba fallecido ex abrupto y que haba muerto ab intestado, cosa que
le comunicaban por si le interesaba denunciar los bienes del difunto y as poder
reclamar su derecho a ser el heredero de su to.
Despus de leer el telegrama, decidi no reclamar su derecho hereditario pensaba- si
mi to abuelo, motu propio no me nombr su heredero, no quiero conseguir por la
fuerza lo que no me dej ad lbitum.
Grosso modo y, mutatis mutandis sta era la personalidad de aquel Jorge Gonzlez,
alias El Galn.
Era la 4:00 p.m. (post merdiem) cuando el personaje de esta historia ficticia se
prepar a retirarse, no sin antes consultar su agenda para recordar las actividades del
siguiente da.

En la narracin anterior se observan ejemplos de locuciones latinas realizadas ex


professo.

20

Las locuciones latinas se usan en espaol principalmente para:


1.- Darle elegancia a un escrito: vale; ex abrupto; alias; consummtum est; ex professo;
ipso facto.
2.- Dar ms fuerza a expresiones:
a) Jurdicas: de jure; de facto; interpsita persona; argumentos ad hminem; infraganti.
b) Filosficas: juicios a priori; juicios a posteriori; ignoti nulla cpido; ex cthedra.
c) Administrativas: ad referndum; dficit; supervit.
3.- Elaborar temas: mens sana in corporis sano; Similla simlibus curntur; Contraria
contraris curntur; Ad astra per spera; aut homo aut nullus.
4.- Usar expresiones contundentes: magster dxit (el maestro ha dicho); prmum
vvere, deinde philosophari (primero debemos vivir, luego filosofar); dura lex, sed lex (la
ley es dura pero es la ley); vox populi, vox Dei (la voz del pueblo es la voz de Dios).
Algunas locuciones latinas forman ya parte de expresiones usadas comnmente
aunque con frecuencia quien las usa no sabe que son expresiones latinas. As:
Agenda
Alias (a)
Ante merdiem (a.m.)
Post merdiem (p. m.)
Et catera (etc.)
Verbi gratiae (vgr.)
Algunas locuciones latinas y su interpretacin:
1.- Ab intestato: sin hacer testamento.
2.- Ad hoc: para esto; especialmente.
3.- Ad lbitum: libremente; voluntariamente.
4.- Ad hminem: al hombre. Un argumento ad hminem es rebatir al contrario con sus
propios argumentos.
5.- Agenda: lo que hay que hacer.
6.- Alias (a): por otro nombre, apodo.
7.- Ante merdiem (a.m.): antes del medioda.
8.- A priori: antes deJuicios a priori, los que se hacen antes de la experiencia.
9.- A posteriori: despus deJuicios a posteriori, despus de la experiencia.
10.- Consummtum est: todo est terminado; ha concluido.
11.- De facto: de hecho.
12.- De jure: De derecho.
13.- Dficit: faltante.
14.- Ex abrupto: bruscamente; algo inesperado.
21

15.- Ex ctedra: desde la ctedra.


16.- Ex professo: a propsito; con toda intencin.
17.- Grosso modo: a grandes rasgos; de manera general.
18.- Jess Nazarenus Rex Iuderum (INRI) Jess Nazareno Rey de los Judos.
19.- Ignoti nulla cpido: lo desconocido no se desea.
20.- In fraganti delicto: en flagrante delito.
21.- Inter nos: entre nosotros; de manera confidencial.
22.- Interpsita persona: persona interpuesta; intermediario.
23.- Ipso facto: por el mismo hecho.
24.- Lapsus linguae: error de lengua, equivocacin al hablar.
25.- Mare mgnum: mar grande; desorden; confusin.
26.- Memorando: recordatorio.
27.- Modus vivendi: manera de vivir.
28.- Mutatis mutandis: cambiando lo que hay que cambiar.
29.- Peccata minuta: pecado pequeo; error sin trascendencia.
30.- Per ccidens: por accidente; por casualidad.
31.- Post merdiem: despus del medioda.
32.- Requiscat in pace (RIP): descanse en paz.
33.- Sui gneris: segn su gnero; muy especial.
34.- Supervit: sobrante; ganancias.
35.- Vale: adis; hasta luego.
36.- Verbi gratia: por gracia de la palabra; por ejemplo.
37.- Id est: es decir.
38.- Loco citato: lugar citado.
39.- Nota bene: ntese bien.
40.- Opus citato: obra citada.
41.- Audaces fortuna jvat: la suerte ayuda a los audaces.
42.- Homo hmini lupus: el hombre es el lobo del hombre.
43.- Nhil nvum sub sole: nada nuevo bajo el sol.
44.- Vvit sub pctore vulnus: la herida vive debajo del pecho.
45.- Cgito, ergo sum: conozco, luego existo.
Algunas sentencias y frases clebres consideradas como locuciones latinas.
1.- Ad astra per spera: hacia los astros a travs de las cosas difciles; a travs de lo
spero. Se deben vencer todas las dificultades para conseguir la superacin.
2.- Ad calendas graecas: para las calendas griegas. Las caldeadas era un da del
calendario romano; el calendario griego no tena calendas. Dejar algo ad calendas
graecas, es posponerlo indefinidamente. Equivale a nuestra expresin: El treinta y uno
de febrero.
3.- urea medicritas: la dorada mediocridad. Esta frase tiene su equivalente en la
expresin In medius est virtus: en el centro, en el equilibrio est la virtud.
4.- Contraria contraris curntur: lo contrario se cura con lo contrario (lema de la
medicina alpata).
22

5.- Gutta cvat lpidem: la gota cava la piedra. Con constancia se consiguen cosas
difciles.
6.- Qui est prior tempore, est ptior jure: quien es primero en tiempo es primero en
derecho.
7.- Similla simlibus curntur: lo semejante se cura
homeopata).

con lo semejante (lema de la

8.- Veni, vidi, vici: llegu, vi, venc; expresin que indica que algo que se crea difcil se
realiz fcilmente.
9.- Vox populi, vox Dei: la voz del pueblo es la voz de Dios. Lema base de la
democracia
10.- Dura lex, sed lex: la ley es dura pero es la ley.
11.- Percata minuta: faltas leves.
12.- Inter nos: entre nosotros

Actividad 5. Elabora un prrafo en donde incluyas algunas locuciones latinas.

______________________________________________________________________
______________________________________________________________________
______________________________________________________________________
______________________________________________________________________
______________________________________________________________________
______________________________________________________________________
______________________________________________________________________
______________________________________________________________________
______________________________________________________________________
______________________________________________________________________
______________________________________________________________________
______________________________________________________________________
______________________________________________________________________
23

ABREVIATURAS BIBLOGRFICAS
A.
A.A.
ap.
apud.
art.
art.cit.
ca., cir.
cap.
cf., cfr, cnfer.
cit.
col.
cuad.
dic.
ed.
edic.
edit.
editl.
e.g.
enc.
fasc.
fig.
fol.
bidem. Ibid.
i.e.
impr.
infla.
Intr.
lm.
l.c., loc. cit.
lib.
ln.
Ms.
Mss.
n.
N.B.
N. del A.
N. del C.
N. del E.
N. del T.
ob. cit., oc., o. cit.
op.
op. cit.
P.
pg.
passim.

autor
autores
aparte, apartado, apndice
que indica que el texto citado se basa en la cita de tal o cual autor
(citado por, apoyado en)
artculo
artculo citado
circa, circter crcum (alrededor de)
captulo
confrntese, comprese
citado
coleccin, columna
cuaderno, cuadrado
diccionario
editor
edicin
editado
editorial
exempli gratia (por ejemplo)
encuadernacin
fascculo
figura
folio
Significa lo mismo. Se utiliza cuando citamos varias veces el
mismo autor sin intercalar citas de otro autor.
id est (es decir)
imprenta
indica vase mas abajo
Introduccin
lmina
loco citato (lugar citado)
libro
lnea
manuscrito
manuscritos
nota
nota bene (ntese bien)
nota del autor
nota del corrector
nota del editor
nota del traductor
obra citada
obra
opus citato (obra citada)
pregunta
pgina
quiere decir indistintamente; en cualquier lugar
24

p. ej.
prol.
pte.
R.
rev.
rst.
s.
s.a.
sec.
s.f.
s.i.
sic.
sig.
s.l.
s.l.f.
s.l.i.
s.q.
s.s.
supra.
t.
tt.
v.
v.g., v.gr.
vid.
vol.
v.v.

por ejemplo
prlogo
parte
respuesta
revista
rstica
siguiente
sin ao
seccin
sin fecha
sin imprenta
significa as, o sea, lo que dice exactamente el autor, aunque sea
confuso o errneo.
siguiente
sin lugar
sin lugar y fecha
sin pie de imprenta
sequentes (siguientes)
siguientes
Para sealar que se ha tratado en el prrafo anterior
tomo
ttulo
vase, verso, versculo
verbi gratia ( por ejemplo)
significa vase, es decir consulte tal o cual obra o texto
volumen
versos, versculo

Actividad 6. Redacta un texto en donde incluyas algunas abreviaturas


bibliogrficas.
______________________________________________________________________
______________________________________________________________________
______________________________________________________________________
______________________________________________________________________
______________________________________________________________________
______________________________________________________________________
______________________________________________________________________
______________________________________________________________________
______________________________________________________________________
______________________________________________________________________
25

ANLISIS CRTICO DE
TEXTOS EXPOSITIVOS

Los textos expositivos se encargan de que una vez terminada una investigacin los
nuevos conocimientos debern de ser expuestos para que otros los conozcan. Esta es
la funcin de las obras expositivas.
Sus pginas contienen informacin cientfica recabada por hombres de ciencia sobre
diversas reas del conocimiento humano.
En estas obras predominan las ideas del autor, comunicando sus observaciones y
reflexiones sobre cualquier fenmeno de la naturaleza u otro aspecto del conocimiento
universal.

PASOS PARA REALIZAR UN ANLISIS CRTICO


DE TEXTOS U OBRAS EXPOSITIVAS
1.- Lectura Estructural o Analtica.
2.- Lectura Interpretativa.
3.- Lectura Crtica o Evaluativa.

26

a) Lectura estructural o analtica: Permite como su nombre lo indica, conocer


cmo est estructurado el libro de texto tanto en su parte externa como interna.
Autor
Ttulo
Editorial

Portada exterior

E
S
T

Anverso

Externa

Autor
Ttulo
Autor de prlogo
Notas
Coleccin

Portada interior

No. de edicin

Reverso

Pie de imprenta

Editorial
Lugar
Fecha

T
U
R
A

Interna

Prlogo
Introduccin Captulos
Notas
Bibliografa
ndices
Colofn

b) Lectura interpretativa: Consiste en atribuir un significado a un texto, despus de


haber ledo y comprendido el vocabulario e ideas principales.
Se hace una buena interpretacin, si despus de haber ledo podemos explicar o
comentar el texto, hacer un resumen o una sntesis.
c) Lectura crtica o evaluativa: Consiste en que despus de haber ledo el texto el
lector est en condiciones de aceptar o rechazar lo que el autor expresa en su obra,
despus de haber reflexionado sobre el tema abordado. Para ello es necesario tener
antecedentes sobre la materia, ciencia o tema de que se trate.

27

ASPECTOS A ANALIZAR EN UNA OBRA EXPOSITIVA


a) Vocabulario: Puede ser tcnico o especializado, con sencillez o de acuerdo con el
nivel acadmico. Si se conoce el vocabulario, eso facilita la comprensin.
Ejemplo:
Texto cientfico
Resulta obvia la necesidad de agua para probar el alimento, convertirlo en una masa
para ser deglutida; las glndulas salivales cubren tal necesidad. Las glndulas mucosas
salivales segregan saliva rica en mucina lubrificante que facilita el paso del alimento al
estmago.

b) Contexto: Cuando el significado de una palabra se deduce por su relacin con los
dems elementos que integran un enunciado, a eso se le llama contexto.
Ejemplo: Para sobresalir en esta empresa se necesita la adhesin de todo el grupo.
La palabra subrayada significa unir, apoyar, cooperar, colaborar, segn su contexto.

c) Tema e ideas: El tema es la idea central de la obra, y en torno a l giran todas las
ideas que lo desarrollan.
Generalmente las ideas se identifican con el ttulo.
Las ideas son las que trata de transmitirnos el autor y se localizan en un inciso, un
prrafo o en todo el captulo.
En el desarrollo de la obra, en cuanto al tema y las ideas, es importante la coherencia
entre los enunciados que integran los prrafos, as como los prrafos que integran un
texto.

28

Esquema general del tema Textos expositivos

CIENTFICOS

Tratan los temas con profundidad.


Utilizan lenguaje tcnico.
Se especula sobre la forma de
aplicar descubrimientos y
estudios cientficos.

TCNICOS
TEXTOS
EXPOSITIVOS

Tienen como objetivo la


tarea de educar.

DIDCTICOS
DIVULGACIN

Su propsito es exponer los


conocimientos al alcance
del lector.
Informan y esclarecen dudas
sobre personajes, lugares o
ramas del conocimiento.

CONSULTA

CLASIFICACIN DE LAS OBRAS O TEXTOS EXPOSITIVOS

Las obras expositivas


clasifican en:
1.- CIENTFICAS
2.- DIDCTICAS
3.- DE DIVULGACIN

29

se

OBRAS CIENTFICAS
Estas obras tratan los temas con gran profundidad. Su lenguaje es tcnico y van
dirigidas a especialistas que por lo general pertenecen o dominan el campo de estudio
de quien escribe el libro, razn por la cual esos textos no pueden ser comprendidos por
cualquier lector.
El uso de trminos o lenguaje especial (tecnicismos) solo permite que la obra sea
comprendida por expertos en la materia que ah se aborde. Esta caracterstica tiene
como consecuencia que estos textos tengan poca difusin y por lo tanto son accesibles
slo para algunos estudiosos.
La obra cientfica es aquella donde se explica en que consisten los misterios de la
naturaleza y sus fenmenos.

EJEMPLO:
En 1747, Benjamn Franklin llam la atencin sobre los efectos maravillosos de los
objetos puntiagudos, tanto para extraer como para lanzar fuego elctrico. (Haba un
desacuerdo sobre qu pararrayos sera mejor: el redondeado o el puntiagudo. Franklin
gan).
Este famoso norteamericano fue el autor de la teora de un fluido que mencionamos
antes. Pero su descubrimiento ms conocido fue el clebre experimento de la cometa.
En aquella poca se crea que las tormentas con rayos eran explosiones de gases en la
atmsfera superior. Pero Franklin, que era un observador muy agudo, not muchas
semejanzas entre los rayos y las descargas elctricas que produca experimentando,
para que fuese slo una coincidencia.

30

Actividad 7. Realiza las siguientes actividades.

a) Conteste los siguientes cuestionamientos.


1. Por qu el lenguaje de las obras cientficas va dirigido a especialistas?
______________________________________________________________________
______________________________________________________________________
______________________________________________________________________
2. Qu palabras del texto son tecnicismos?
______________________________________________________________________
3. A qu rea de la ciencia pertenece el texto?
______________________________________________________________________
4. A quin le puede interesar el texto anterior?
______________________________________________________________________
5. Cules son las ideas principales del texto?
______________________________________________________________________
______________________________________________________________________
b) De las siguientes palabras subraya las que consideres que son de un lenguaje
especializado y seala a qu campo pertenecen:
METAMORFOSIS CIELO HBITAT
MESA

ESCRITORIO

EVOLUCIN

EXTINCIN HOGAR

ESPECIES

FOTOSNTESIS

Pertenecen al campo de: _________________________________

31

OBR AS DID CTIC AS


Las obras didcticas tienen como objetivo primordial la tarea de educar.
Pretenden ensear de manera gradual los diversos conocimientos de un tema o
materia determinada.
Presentan los conocimientos cientficos, tcnicos y humansticos explicados con
un lenguaje sencillo y de acuerdo con el nivel acadmico del estudiante, para
que ste lo asimile con facilidad.
Ejemplo de estas obras son los libros que existen en las instituciones educativas,
desde primaria hasta la universidad, o cualquier otra donde se impartan
conocimientos.

EJEMPLO:
FUNCIN PROPOSICIONAL
El primer concepto que necesitamos entender es el de funcin proposicional.
Vamos a partir de esta proposicin:
Mxico es un pas de Amrica.
De ella podemos decir que tiene sentido, enuncia algo y es verdadera.
Si de la misma suprimimos el sujeto, entonces quedara as:
_______________es un pas de Amrica.
En esta expresin aparece un lugar vaco: el sujeto, y por esta razn no tiene sentido,
no es proposicin, ni se puede considerar como verdadera o falsa.
A una proposicin de este tipo se le llama funcin proposicional.

32

Actividad 8. Contesta las siguientes cuestiones.

1. Qu fue lo que aprendiste en el ejemplo anterior?


______________________________________________________________________
______________________________________________________________________
______________________________________________________________________
2. Escribe el nombre de obras didcticas que utilizas actualmente.
______________________________________________________________________
______________________________________________________________________
3. Cules son sus caractersticas?
______________________________________________________________________
______________________________________________________________________

*Despus

del ejemplo anterior cabe aclarar que existen textos literarios que tambin
tienen una funcin didctica ya que de alguna manera, en ocasiones, tambin nos
proporcionan una enseanza o moraleja como en el caso de las fbulas.

Ejemplos de Fbula:
Los hijos desunidos del labrador
Un padre tena varios hijos que peleaban constantemente entre s. Al darse cuenta de
que era imposible sanar las disputas con sus exhortaciones, decidi darles un ejemplo
prctico sobre los males de la desunin. Para este propsito, un da les pidi que le
llevaran un atado de varas.
Cuando lo hicieron, el padre coloc el haz en las manos de cada uno de ellos y, uno a
uno, les orden que lo rompieran en pedazos. Los hijos trataron con todas sus fuerzas y
ninguno tuvo xito. Acto seguido, el padre desat el haz, tom las varas y las distribuy
entre sus hijos. Y as pudieron romperlas con facilidad.
Entonces se dirigi a ellos y les dijo: Queridos hijos mos, si ustedes comparten
sus opiniones y se unen para ayudarse entre s, sern como el haz de varas y sus
enemigos no podrn hacerles dao; pero si dejan que la discordia se apodere de
sus vidas, acabarn por romperlos con la misma facilidad.

33

Esopo
El Buey y el mosquito
En el cuerno de un buey se pos un mosquito.
Luego de permanecer all largo rato, al irse a su vuelo pregunt al buey si se alegraba
que por fin se marchase.
El buey le respondi:
- Ni supe que habas venido. Tampoco notar cuando te vayas.
Pasar por la vida, sin darle nada a la vida, es ser insignificante.
Esopo

OBRAS DE DIVULGACIN
Las obras de divulgacin exponen los conocimientos de manera ligera, simple y amena
ponindolas al alcance de cualquier lector.
Su objetivo es hacerlo partcipe de todos los avances que se van desarrollando en las
diversas especialidades del saber humano.

34

Van dirigidas a personas


que no son estudiantes de
un rea determinada.
Estn escritas con un
lenguaje claro y conciso,
al alcance de cualquier
lector, que desee
incrementar sus
conocimientos o tener una
visin ms general de lo
que ocurre a su alrededor.

EJEMPLO:
Hacia los aos sesenta del siglo pasado, el qumico francs Louis Pasteur propuso la
teora germinal de las enfermedades, segn la cual, todas las enfermedades
contagiosas eran causadas y propagadas por alguna forma diminuta de vida que se
multiplicaba en el organismo enfermo, pasaba de ese organismo a otro sano, lo haca
enfermar, y as sucesivamente.
Pasteur, sin embargo, estaba trabajando con una enfermedad mortal: la rabia, (tambin
llamada hidrofobia), y descubri que aunque la enfermedad era contagiosa y poda
contraerse por el mordisco de un animal rabioso, no se vea el germen por ningn lado.
Pasteur concluy que el germen s que estaba ah, pero que era demasiado pequeo
para verlo con el microscopio con que trabajaba.

Actividad 9. Contesta las siguientes cuestiones.

1. Qu avance cientfico descubri Luis Pasteur?


______________________________________________________________________
______________________________________________________________________
______________________________________________________________________
2. En qu medios aparece informacin de divulgacin?

35

______________________________________________________________________
______________________________________________________________________
3. Escribe el nombre de publicaciones que conozcas.
______________________________________________________________________
______________________________________________________________________
4. Qu temas te gusta leer en las obras de divulgacin?
______________________________________________________________________
______________________________________________________________________

ANLISIS DE
TEXTOS
INFORMATIVOS

El periodismo, como texto informativo que es, surge de una necesidad intrnseca del ser
humano de conocer lo que sucede en su entorno, y a la vez comunicarlo, pues es un
ser consciente que, al percatarse de los hechos que ocurren, es capaz de intervenir en
su momento histrico y dejar su huella plasmada en l.

El periodismo es una disciplina cuyo objetivo es informar de los hechos ms


sobresalientes que ocurren en distintas partes del mundo y que interesan a una
colectividad. En l se tratan asuntos de la vida nacional o internacional. Su temtica
puede versar sobre hechos de carcter econmico, poltico, cultural o artstico.

El periodismo como disciplina pertenece a un amplio campo de interacciones de


comunicacin, cuyos objetivos son: la noticia, las fuentes de informacin, la opinin
pblica, la objetividad y subjetividad en el manejo de la informacin y, por ende, la tica
periodstica y la libertad de prensa y tambin de manera relevante los efectos que los
mensajes periodsticos tengan en los receptores.

36

El periodismo es una parte de la comunicacin humana destinada a la transmisin de


noticias y cuyas caractersticas son:

Actualidad
Universalidad
Periodicidad
Acceso pblico
Objetividad

EL PERIDICO
En cuanto a la estructura del peridico, existen dos tamaos:
1.- El clsico o estndar: de mayor dimensin, ejemplos: El Norte, El Porvenir, Exclsior
y El Universal.
2.- El tabloide o chico: ejemplos La Jornada, El Metro, El Milenio y La Prensa.
En la presentacin del diario, que es la primera hoja, encontramos:
1.- El ttulo del peridico que se representa con un logotipo a manera de distintivo.
2.- Cerca del ttulo aparece el lema, que en forma breve pretende enunciar el objetivo
general de la publicacin, adems de llamar la atencin del pblico.
Ejemplos de lemas:
Alarma: El diario rojo de Mxico.
Exclsior: El peridico de la vida nacional.
El Da: El peridico para leerse da con da.
3.- Despus del lema viene el fechario, donde adems de la fecha se consigna el
nmero del peridico, ao, tomo o volumen y en algunos casos nombres de directorio o
personal que est al frente de la publicacin.
4.- El directorio es el espacio donde se incluyen los nombres del grupo responsable de
la publicacin por orden de responsabilidad.
Otros elementos que lo conforman son:

Encabezado: Se enuncia la noticia ms relevante del da, de lado a lado de la


primera plana, es decir a 8 columnas en el tamao clsico, y de 5 6 en el
tamao tabloide.
Entrada o Lead: Es el resumen inicial donde se da lo esencial de la informacin;
puede constar de una palabra o frase y sirve para captar la atencin del lector.
Cintillo: Se ocupa de la segunda noticia en importancia.
Oreja: Es la parte que resalta o sobresale y que incluye el nombre del peridico
y su tema correspondiente.
Sumario: Hace mencin de los temas centrales en la informacin.
37

Fotografas: Sirven para ilustrar y hacer ms atractiva la presentacin; van


acompaadas de un pie de grabado que alude a lo representado en la fotografa.

Por otro lado, la organizacin de las publicaciones periodsticas determina que se


dividan en secciones, segn el contenido. Las secciones se presentan en varios
conjuntos de hojas, dobladas por separado, y en la parte superior de la hoja se indica a
qu seccin pertenece de acuerdo con el asunto que trate; con esto se facilita el
manejo y se permite un acceso rpido a la informacin necesaria.
Las secciones en las que se divide un peridico segn la temtica general son:

Seccin de noticias nacionales e internacionales


Seccin de noticias culturales
Seccin de noticias sociales
Seccin de espectculos
Seccin deportiva
Seccin burstil
Seccin policaca
Avisos clasificados
Tiras cmicas y pasatiempos

El tiraje es el nmero de ejemplares que se editan por ocasin; es decir, cada vez que
se produce (an cuando sea diariamente).

Actividad 10. Selecciona una seccin del peridico y redacta una noticia.

38

LA

NOTICIA

En un peridico la noticia o nota informativa es la que aparece con mayor


frecuencia, razn por la cual se le puede considerar como el alma del peridico.

Dar a conocer una noticia es igual que informar; es lo que se necesita saber y sacia la
curiosidad de la gente. Es el elemento esencial del periodismo.
Ahora bien, con un enfoque profesional como se maneja dentro del periodismo actual,
la noticia, adems de despertar el inters del lector y satisfacer su curiosidad, debe de
expresarse de manera correcta, breve y clara.
Dentro de la tcnica adecuada para la redaccin de noticias, existen algunos
cuestionamientos que se han inferido del quehacer periodstico y que manejan varios
estudiosos de la materia y que nos auxilian para comprender una noticia:

Quin? Sujeto de la informacin.


39

Qu? El hecho, lo que ha sucedido.


Cmo? Es el mtodo o manera de producirse el hecho.
Dnde? El sitio o lugar en que sucede el acontecimiento.
Cundo? Factor tiempo (ao, da, hora o minuto). La precisin de la fecha
depende del hecho.
Por qu? La causa, elemento fundamental que nos da la razn de lo que ha
pasado.

Si el periodista responde a estas cuestiones, la noticia quedar planteada


correctamente en toda su dimensin, siempre y cuando se parta de lo ms relevante y
significativo de un acontecimiento. Esto atraer hacia ella la atencin del pblico.

Actividad 11. Redacta una noticia de algn acontecimiento relevante de tu escuela


o tu localidad. Considerando las caractersticas antes mencionadas.

40

GNEROS PERIODSTICOS
Tanto en los peridicos como en las revistas, se pueden incluir varios de los siguientes
gneros periodsticos:
Noticias: Se trata de eventos de inters general que acaban de suceder.
Notas: Se trata de noticias cortas, breves, que no requieren de tanta explicacin.
Entrevistas: Se presenta la opinin de un personaje de la vida poltica, social,
econmica, deportiva o del espectculo, sobre algn tema de actualidad.
Encuestas: Equivalen a entrevistas cortas repetidas un gran nmero de veces a
diferentes personas. Se orientan sobre todo a descubrir la opinin pblica sobre
determinados sucesos o acontecimientos.
Reportajes: Investigaciones a fondo sobre temas especficos de actualidad.
Incluyen antecedentes histricos de la situacin o del problema, descripcin de la
realidad actual, noticias, entrevistas, comentarios y conclusiones sobre ese
problema. Hay reportajes demostrativos (o comprobatorios), descriptivos,
narrativos, especializados en un rea o tema de la vida actual, profundos, de
entretenimiento y grficos. Estos ltimos se utilizan sobre todo en el cine y la
televisin.
Crnicas: Son relatos detallados de un suceso actual o actualizado siguiendo
una secuencia cronolgica.
Columnas: Son espacios predeterminados y fijos en el peridico o en la revista,
propiedad de un autor, con un nombre fijo, y que tratan temas habituales.
41

Crticas: El autor d su punto de vista crtico, puede ser positivo o negativo,


sobre acontecimientos deportivos, artsticos y/o literarios.
Ensayos: Es el punto de vista personal y analtico que asume el autor al abordar
un tema determinado de manera intensa y metdica. Son estudios y monografas
que tienen ms cabida en revistas que en peridicos.
Artculos: El autor juzga y critica lo ocurrido. Puede tratarse de artculos
firmados por el autor, o editoriales sin firma, que son responsabilidad oficial del
peridico que los publica.
Documentos: Es la publicacin de discursos, ponencias, mensajes,
conferencias resmenes o captulos de libros, etc. Pueden publicarse
ntegramente en una sola edicin, o irse publicando por partes, si son muy
extensos.

Actividad 12.
Trabajo en equipo:
a) Realicen un peridico escolar.
b) Contesten las siguientes cuestiones.
1.- Qu es el periodismo?
______________________________________________________________________
______________________________________________________________________
2.- Qu es un peridico?
______________________________________________________________________
______________________________________________________________________
3.- Cules son las secciones de un peridico?
______________________________________________________________________
______________________________________________________________________
4.- Qu es noticia?
______________________________________________________________________
______________________________________________________________________
5.- Cules son las interrogantes que auxilian para comprender una noticia?
______________________________________________________________________
______________________________________________________________________
6.- Explica los gneros periodsticos:
42

______________________________________________________________________
______________________________________________________________________
______________________________________________________________________
______________________________________________________________________
______________________________________________________________________
______________________________________________________________________
______________________________________________________________________
______________________________________________________________________
______________________________________________________________________
______________________________________________________________________
______________________________________________________________________

ANLISIS
DE TEXTOS
LITERARIOS

T
E
X
T
O
S
L
I
T
E
R
A
R
I
O
S

NARRATIVO

Cuento
Novela

DRAMTICO

Tragedia
Comedia

LRICO

Himno
Oda
Elega
Cancin

43

OBRAS

NARRATIVAS

Narrar algo significa contar o relatar una o varias acciones, hechos o sucesos que nos
ocurrieron a nosotros mismos o a otras personas. Tambin se pueden narrar
acontecimientos que no sucedieron realmente, pero que uno lo imagina o lo inventa.
Obras Narrativas:
Serie de actos o expresiones
verbales mediante los cuales una
persona relata un suceso.

Son tres los componentes principales de una narracin:


1. El ambiente: Situacin en la que se di el hecho que se va a relatar.
2. Los personajes: Son quines intervienen en la narracin.
3. La accin: Es lo que sucedi, la forma como intervino cada personaje, as como
las reacciones que stos presentaron ante lo sucedido.
Las narraciones pueden ser de varios tipos: cuentos, novelas, fbulas, leyendas y
ancdotas. Las que aprenderemos a continuacin son el cuento y la novela.

El Cuento
El cuento es el gnero ms antiguo y popular, es una expresin de los pueblos
primitivos.

Cuento:
Consiste en una narracin corta, que
trata de un solo tema y que presenta un
argumento completo en s mismo, en el
que se incluyen unos cuantos
personajes.

Para elaborar un cuento se debe definir y describir los siguientes elementos:


44

1. Los personajes: Pueden aparecer en forma directa o indirecta segn lo decida el


autor.
2. La situacin o ambiente: Es el lugar fsico en donde se realiza la accin.
3. El tiempo: Deber coincidir con la poca en que se ambienta la historia y con la
duracin del suceso que se narra.
4. Trama: Es el conflicto que mueve la accin del relato y el motivo de la narracin.
5. El tono: Equivale a la actitud que adopta el autor en su obra, el cual puede ser
humorstico, alegre, irnico, sarcstico, etc.
6. El desenlace o el final.
Todo cuento deber tener unidad narrativa, esto es: introduccin, exposicin,
desarrollo o nudo y desenlace o desenredo.

Ejemplo:
Una vez, en la antigua India, haba un gigante malo que posea un magnfico castillo
con vistas al mar. Como el gigante haba estado fuera muchos aos guerreando, los
nios del pueblo cercano solan ir a jugar al hermoso jardn del gigante. Un da, el
gigante regres y ech de su jardn a todos los nios. No quiero verlos ms aqu!, buf
mientras cerraba con estruendo la gran puerta de roble. Luego levant un enorme muro
de mrmol en torno al jardn para que no entraran los nios. Lleg el invierno, con el fro
que es habitual en las zonas septentrionales, y el gigante ansiaba que volviera el calor.
La primavera ilumin el pueblo que haba a los pies del castillo, pero las fras garras del
invierno no abandonaron su jardn. Un da, el gigante percibi por fin las fragancias
primaverales y not que el sol entraba radiante por sus ventanas. Por fin la primavera!,
exclam, corriendo al jardn. Pero no estaba preparado para lo que vi. Los nios del
pueblo haban conseguido escalar la pared del castillo y estaban jugando en el jardn.
Era debido a su presencia que el jardn se haba transformado en un lugar exuberante
poblado de rosas, margaritas y orqudeas. Todos los nios rieron de jbilo, excepto uno,
que era mucho ms bajo que los dems. Lloraba con desconsuelo pues no tena fuerza
suficiente para saltar el muro y jugar en el jardn. El gigante sinti lstima y, por primera
vez en su vida, se arrepinti de su maldad. Ayudar a ese nio, dijo, corriendo hacia
l. Cuando los otros lo vieron venir, huyeron del jardn temiendo por sus vidas. Pero el
ms pequeo se mantuvo firme. Yo matar al gigante. Defender nuestro lugar de
recreo, dijo. Cuando el gigante se acerc al nio, abri sus brazos y le dijo: He venido
a ayudarte a saltar el muro para que juegues en el jardn. A partir de ahora ser tuyo.
El nio, convertido en hroe, se sinti muy feliz y regal al gigante el collar de oro que
siempre llevaba al cuello. Es mi amuleto de la suerte. Quiero que lo lleves t dijo.
Desde aquel da, los nios jugaron con el gigante en el jardn del castillo. Pero aquel
valiente muchacho, que era el preferido del ogro, ya no volvi. Con el tiempo, el gigante
enferm y se debilit. Los nios seguan jugando en el jardn pero l ya no tena fuerzas
para estar con ellos. En aquellos das, el gigante no pensaba ms que en aquel
muchacho. Un da de invierno especialmente crudo, el gigante mir por su ventana y vio
algo milagroso: aunque la mayor parte del jardn estaba cubierta de nieve, en mitad del
mismo haba un estupendo rosal rebosante de flores espectaculares. Junto a las rosas
estaba el nio en quien el gigante haba pensado tanto. El muchacho sonrea
45

dulcemente. El gigante corri a abrazar al nio. Dnde has estado todos estos aos,
mi joven amigo? Te he echado muchsimo de menos. El muchacho dio una respuesta
meditada: Hace mucho tiempo me ayudaste a entrar en el jardn mgico. Ahora he
venido para que entres t en el mo. Ms tarde, cuando los otros nios fueron a ver al
gigante, lo hallaron inerte en el suelo. Estaba cubierto de pies a cabeza por millares de
bellas rosas.

Tomado del libro El Monje que Vendi su Ferrari


Autor: Robin S. Sharma

Actividad 13. Inventa un cuento y escrbelo sobre las lneas.

______________________________________________________________________
______________________________________________________________________
______________________________________________________________________
______________________________________________________________________
______________________________________________________________________
______________________________________________________________________
______________________________________________________________________
______________________________________________________________________
______________________________________________________________________
______________________________________________________________________
______________________________________________________________________
______________________________________________________________________
______________________________________________________________________
______________________________________________________________________
______________________________________________________________________
______________________________________________________________________
______________________________________________________________________
______________________________________________________________________
46

______________________________________________________________________
______________________________________________________________________
______________________________________________________________________
______________________________________________________________________
______________________________________________________________________
______________________________________________________________________
______________________________________________________________________
______________________________________________________________________
______________________________________________________________________
______________________________________________________________________
______________________________________________________________________
La Novela
Novela:
Es un escrito de dimensiones
considerables que se redacta en prosa
sobre acontecimientos ficticios.

La novela presenta mayor extensin en contenido, incluye un mayor nmero de


personajes y relata un mayor nmero de sucesos o acciones, en diferentes ambientes o
situaciones. A lo largo de la novela se pueden ir tratando diferentes argumentos, los
cuales se van cerrando o completando conforme avanza la historia.
Las novelas (tanto las escritas, como las de radio y televisin) se presentan en
captulos. El primer captulo presenta a los personajes principales de la historia y
empieza a describir uno o varios argumentos que ir tratando. En cada captulo
subsecuente, se van cerrando algunos de los argumentos planteados anteriormente, y
se van abriendo nuevas situaciones, de tal forma que la historia mantiene el suspenso
hasta el final de la misma. En el ltimo captulo deben quedar cerrados todos los
argumentos que se fueron planteando a lo largo de la novela, a no ser que el autor est
pensando en escribir una segunda parte.

47 novela y la expondrn ante el grupo.


Actividad 14. En equipos seleccionarn una

Actividad 15. Contesta las siguientes preguntas.

1. En qu consisten las obras narrativas?


______________________________________________________________________
______________________________________________________________________
2. Qu significa narrar?
______________________________________________________________________
______________________________________________________________________
3. Cules son y en qu consisten cada uno de los tres componentes principales de
una narracin?
______________________________________________________________________
______________________________________________________________________
______________________________________________________________________
4. Cules son los tipos de narraciones que hay?
______________________________________________________________________
5. En que consiste el cuento?
______________________________________________________________________
______________________________________________________________________
6. Escribe los elementos que debes tomar en cuenta para elaborar un cuento.
______________________________________________________________________
7. Qu significa que el cuento tenga unidad narrativa?
______________________________________________________________________
______________________________________________________________________
8. Qu es la novela?

48

______________________________________________________________________
______________________________________________________________________
______________________________________________________________________
9. Qu diferencia existe entre la novela y el cuento?
______________________________________________________________________
______________________________________________________________________
______________________________________________________________________
10. Cul es la forma en que se presentan las novelas?
______________________________________________________________________

OBRAS

DRAMTICAS

Obra Dramtica:
Se caracteriza por presentar sus temas en
forma dialogada. Tratan asuntos relacionados
con las pasiones humanas en sus diferentes
facetas y son representados por personajes
que crea el mismo autor.

El esquema estructural de la obra dramtica est compuesta por:


1. Exposicin: Son los supuestos de la accin, es decir, la situacin de partida, la
primera caracterizacin de los personajes.
2. Conflicto: Es un tipo de problema que nace cuando se enfrentan los deberes, por
un lado, y los deseos, sentimientos, tendencias, por el otro.
3. Clmax: Es el punto culminante de la accin dramtica.
4. Resolucin: Es el desenlace, cuando se solucionan los conflictos.

Las caractersticas de las obras dramticas son las siguientes:


a) Escenografa: Es una obra literaria producida por un dramaturgo y es susceptible
de llevarse a escena por los actores. Recibe un gran realce con ayuda de luces,
vestuario, decorado, etctera.
b) Personajes dramticos: Est compuesto por las personas que intervienen en la
obra.
c) Dilogo: La forma idnea de expresin en una obra dramtica es el dilogo, la
palabra viva.
49

d) Conflicto: En los dilogos de los personajes stos emiten sus puntos de vista de
los cuales el autor se vale para abordar distintos argumentos e intentar
persuadir a los lectores o a los espectadores para tomar una posicin.
e) Tensin hacia un desenlace: Est compuesto por los sucesos de mayor tensin,
los cuales son el ncleo de la obra dramtica.
Dentro del gnero dramtico las obras ms representativas son: la tragedia, la
comedia, la tragicomedia, el melodrama, etctera. Las que estudiaremos a continuacin
sern la tragedia y la comedia.

La Tragedia
Tragedia:
Es aquella en la que el autor escribe su
obra de tal manera que provoca
en el auditorio la piedad y el terror,
al fin de que al mezclarse ambas
emociones venga la catarsis.

De acuerdo con la teora de Aristteles, la tragedia perfecta debe cumplir con los
siguientes requisitos:
El autor escribe provocando piedad y terror en el auditorio.
La destruccin del hroe trgico est causada por un error de la fatalidad o por
la decisin de los dioses.
La idea de nobleza es fundamental en la vida del hroe, inexorablemente
condenado al infortunio y a la muerte.
El hroe asumir su despiadado destino con dignidad, pero protestando
vigorosamente contra la condena divina.
La inmensa capacidad de sufrimiento que el autor dar a su protagonista le
permitir reconocer la magnitud de su sacrificio convirtindose en un ser
equilibrado y sabio, despojada de la arrogancia que le mereci la condena de los
dioses.

50

Los tipos trgicos creados en Grecia no slo han permanecido, sino que han
trascendido a otras reas no artsticas, como la psicologa y la astronoma.
La tragedia est formada por: el prlogo, episodios y el xodo o desenlace.

La Comedia

Comedia:
Es la representacin popular de un suceso
cmico mezclado con mmica, burlas,
farsas, alusiones personales y
desenfrenada crtica poltica.

La comedia naci en las fiestas de vendimia, en las que el placer se manifestaba


con locura y alegra. Parte principal de estas fiestas era el Komos, festn animado y
bullicioso, sazonado con picarescos chistes y canciones de sobremesa.
Aristfanes es el principal representante de la comedia antigua, ya que
estableci sus rasgos bsicos y universales, a saber:
1.
2.
3.
4.
5.
6.

Stira violenta.
Polmica poltica.
Alusin a personajes destacados.
Argumento que narre un hecho gracioso.
Conocimiento de la naturaleza humana.
Expresiones chispeantes.

51

Actividad 16. Contesta las siguientes preguntas.

1. En que consisten las obras dramticas?


______________________________________________________________________
______________________________________________________________________
2. Cul es el esquema estructural de la obra dramtica?
______________________________________________________________________
______________________________________________________________________
______________________________________________________________________
______________________________________________________________________
3. Cules son las caractersticas de las obras dramticas?
______________________________________________________________________
______________________________________________________________________
______________________________________________________________________
______________________________________________________________________
______________________________________________________________________
4. Cules son las obras representativas del gnero dramtico?
______________________________________________________________________
5. Qu entiendes por tragedia?
______________________________________________________________________
______________________________________________________________________
______________________________________________________________________
6. Cules son los requisitos de la tragedia?
______________________________________________________________________
______________________________________________________________________
______________________________________________________________________
______________________________________________________________________
7. Cmo est estructurada la tragedia?
______________________________________________________________________
______________________________________________________________________
8. Qu es la comedia?
______________________________________________________________________
______________________________________________________________________
9. Quin es el principal representante de la comedia antigua?
______________________________________________________________________

52

10. Cules son los rasgos bsicos y universales de la comedia?


______________________________________________________________________
______________________________________________________________________
II. Escribe las diferencias que existen entre la tragedia y la comedia.
______________________________________________________________________
______________________________________________________________________
______________________________________________________________________
______________________________________________________________________
______________________________________________________________________
______________________________________________________________________
______________________________________________________________________
______________________________________________________________________
______________________________________________________________________
______________________________________________________________________

OBRAS

LRICAS

Obras Lricas:
En estas obras la subjetividad alcanza
su mxima expresin,
pues la idea primordial del autor es
transmitir el cmulo de emociones y
sentimientos que lo embargan al
escribir sus vivencias.

Las obras lricas contienen figuras literarias que enriquecen enormemente su


contenido y deleitan al lector que tiene la oportunidad de leerlas. Estos textos implican
sentimientos, emociones o deseos, escritos en verso o en prosa potica, con ellos el
autor manifiesta su yo interno directamente o por medio de un personaje determinado.
Dentro de este gnero literario destacan: el himno, la oda, la elega, la cancin, la copla,
etc.

El Himno
Himno:
Es una composicin con texto de
alabanza que expresa sentimientos
de fervor religioso o patritico.
53

Los versos del himno se deben acomodar por su regularidad y ritmo a los
compases de la msica.
Los himnos ms antiguos que se conservan son dos ejemplos de la antigua
Grecia dedicados al Dios Apolo, descubiertos en Delfos, y que datan del siglo II a.C.
Otras civilizaciones de la antigedad como la China, la Egipcia y la India han dejado
documentos con algunos himnos aunque no con su msica. El canto de himnos dentro
del judasmo y el cristianismo data al menos de la poca del libro bblico de los Salmos,
cuyo nombre en hebreo, Tehillim, significa 'canciones de alabanza'. Slo se conservan
algunas melodas cristianas de canto llano antiguo, la ms antigua de las cuales data
de alrededor del 300 d.C.
La Oda
Oda:
Tiene por objeto cantar los ms
puros sentimientos del alma, en medio
de arrebatos de admiracin y
entusiasmo, o con delicadas expresiones
de ternura y amor.

Las odas eran originalmente poemas para ser cantados con el acompaamiento de un
instrumento musical. De tono elevado, estaba destinada a exaltar la vida de un
individuo, a conmemorar un hecho importante o a describir la naturaleza de manera
ms intelectual que emocional.
Las odas se dividen por lo general, en religiosas, heroicas, morales o filosficas y
gratulatorias.
La oda clsica exiga que el poeta manifestara algunos rasgos de su
personalidad como prueba de vehemencia con la cual intentaba aplacar a los dioses.
Posteriormente, cuando la oda pas a otras culturas, adopt formas variadas acordes a
los distintos intereses.
Entre los poetas contemporneos autores de odas y sus variantes se
encuentran Pablo Neruda, Federico Garca Lorca y Jorge Luis Borges.
ODA AL LIBRO (II)
Libro hermoso, libro, mnimo bosque, hoja tras hoja, huele tu papel a elemento, eres matutino y
nocturno, cereal, ocenico, en tus antiguas pginas cazadores de osos, fogatas cerca del
Mississippi, canoas en las islas, ms tarde caminos y caminos, revelaciones, pueblos

54

insurgentes, Rimbaud como un herido pez sangriento palpitando en el lodo, y la hermosura de


la fraternidad, piedra por piedra sube el castillo humano, dolores que entretejen la firmeza,
acciones solidarias, libro oculto de bolsillo en bolsillo, lmpara clandestina, estrella roja.
Nosotros los poetas caminantes exploramos el mundo, en cada puerta nos recibi la vida,
participamos en la lucha terrestre. Cul fue nuestra victoria? Un libro, un libro lleno de
contactos humanos, de camisas, un libro sin soledad, con hombres y herramientas, un libro es
la victoria. Vive y cae como todos los frutos, no slo tiene luz, no slo tiene sombra, se apaga,
se deshoja, se pierde entre las calles, se desploma en la tierra.
Libro de poesa de maana, otra vez vuelve a tener nieve o musgo en tus pginas para que las
pisadas o los ojos vayan grabando huellas: de nuevo descrbenos el mundo los manantiales
entre la espesura, las altas arboledas, los planetas polares, y el hombre en los caminos, en los
nuevos caminos, avanzando en la selva, en el agua, en el cielo, en la desnuda soledad marina,
el hombre descubriendo los ltimos secretos, el hombre regresando con un libro, el cazador de
vuelta con un libro, el campesino arando con un libro.
Pablo Neruda

La Elega

Elega:
Obras que lamentan los
acontecimientos funestos
y expresan la melancola del alma.

Las elegas clsicas eran a menudo cantos nostlgicos; composiciones que denotaban
lamentaciones por diversas causas, pero las hay tambin amorosas, religiosas,
patriticas y fundamentalmente funerales.
En la poesa moderna (desde el siglo XVI), las elegas se caracterizan por su
contenido melanclico y centrado en la muerte.

La Cancin

Cancin:
Es un pequeo poema tomado de la
literatura del renacimiento.
Su carcter es siempre tierno y delicado.
El tono suele ser elevado y optimista.

55

La cancin es un texto corto, lrico o narrativo, acompaado de msica. La msica a


menudo reproduce el sentimiento del texto e intenta subrayar su contenido emocional.
Suele ser un poema con msica. En su acepcin moderna, el trmino cancin suele
limitarse a las composiciones para una o dos voces, frecuentemente con
acompaamiento instrumental. En Occidente invariablemente estn forjadas en formas
de repeticiones de versos como la balada, en la que se usa una nica meloda una y
otra vez para poner msica a una serie de estrofas.

Actividad 17. Escribe una cancin atendiendo al gnero lrico.

______________________________________________________________________
______________________________________________________________________
______________________________________________________________________
______________________________________________________________________
______________________________________________________________________
______________________________________________________________________
______________________________________________________________________
______________________________________________________________________
______________________________________________________________________
______________________________________________________________________
______________________________________________________________________
______________________________________________________________________
______________________________________________________________________
______________________________________________________________________
______________________________________________________________________
______________________________________________________________________
______________________________________________________________________
______________________________________________________________________

56

______________________________________________________________________
______________________________________________________________________
______________________________________________________________________
______________________________________________________________________
______________________________________________________________________
______________________________________________________________________
______________________________________________________________________
______________________________________________________________________
______________________________________________________________________________
______________________________________________________________________________
______________________________________________________________________________
Actividad 18. Contesta las siguientes preguntas.

1. Qu son las obras lricas?


______________________________________________________________________
______________________________________________________________________
2. Cules son las obras representativas del gnero lrico?
______________________________________________________________________
3. En qu consiste el himno?
______________________________________________________________________
4. Qu es la oda?
______________________________________________________________________
______________________________________________________________________
5. Cmo se dividen las odas?
______________________________________________________________________
6. Cules son algunos poetas contemporneos autores de odas?
______________________________________________________________________
7. Escribe una Oda.
______________________________________________________________________
______________________________________________________________________
______________________________________________________________________
______________________________________________________________________
______________________________________________________________________
______________________________________________________________________
57

______________________________________________________________________
______________________________________________________________________
______________________________________________________________________
______________________________________________________________________
8. Qu es la elega?
______________________________________________________________________
______________________________________________________________________
9. Cules son las clases de elegas que existen?
______________________________________________________________________
______________________________________________________________________
10. En qu consiste la cancin?
______________________________________________________________________
______________________________________________________________________
Actividad 19. Redacta una composicin de una obra lrica.

______________________________________________________________________
______________________________________________________________________
______________________________________________________________________
______________________________________________________________________
______________________________________________________________________
______________________________________________________________________
______________________________________________________________________
______________________________________________________________________
______________________________________________________________________
______________________________________________________________________
______________________________________________________________________
______________________________________________________________________
______________________________________________________________________
______________________________________________________________________
______________________________________________________________________
______________________________________________________________________
______________________________________________________________________
______________________________________________________________________
______________________________________________________________________
58

______________________________________________________________________
______________________________________________________________________
______________________________________________________________________
______________________________________________________________________
______________________________________________________________________
______________________________________________________________________
______________________________________________________________________
______________________________________________________________________
______________________________________________________________________
______________________________________________________________________

U
N
I
D
A
D
59

Expresin Escrita
TRABAJO
ACADMICO

Todo alumno tiene como base de su ciclo de estudios la necesidad de elaborar


mltiples trabajos acadmicos de investigacin; de ah que sea necesario conocer
todos los pasos para la elaboracin de los mismos. Esto permitir al estudiante
ejercitar su habilidad de manera sistemtica enriqueciendo el significado del curso.
Etimolgicamente el vocablo investigar proviene del latn investigo as are, que
significa seguir la pista o la huella de algo.
Investigar es indagar, averiguar, buscar, es descubrir lo que estaba oculto.
Son muy variables las modalidades que adopta un trabajo de investigacin escolar, las
ms usuales son las siguientes:
A)
B)
C)
D)
E)

Informes
Ensayos
Resea
Monografas
Investigaciones
60

F) Tesis
Los hechos observados dentro de una investigacin forman la metodologa que
constituye el procedimiento o serie de pasos
que nos conducen a obtener
conocimientos.

TIPOS DE INVESTIGACIN
En referencia al mtodo que se utiliza, la investigacin se divide en tres ramas:
La Investigacin es Documental: Cuando la informacin se recopila y se centra en
documentos (libros, actas, revistas, o cualquier otro medio impreso).

La Investigacin De Campo: Se realiza en el medio ambiente en donde se desarrolla el


problema, en el sitio en que ocurre el fenmeno; el investigador interviene en ellas ya
sea como observador, encuestador, entrevistador o como actor participante.

61

La Investigacin Experimental de laboratorio: Se lleva a cabo en un recinto cerrado,


en los que existe un estricto control de las variables y del ambiente en el que ocurre el
fenmeno. La Investigacin de Observacin en las Ciencias Sociales es cuando el
experimento es con grupos humanos para observar sus reacciones.

Actividad 1: A continuacin se te presentan algunas cuestiones. Contesta


segn sea el caso utilizando las siguientes opciones:
Investigacin Documental
Investigacin De Campo
Investigacin Experimental
1. Conocer la vida y obra de Benito
Jurez
____________________________________
2. Encontrar un nuevo medicamento
para la gripe.
____________________________________
3. Conocer el comportamiento de las
personas en un mercado.
____________________________________
4. Conocer todo sobre la Historia de
Mxico.
____________________________________
5. Saber las actitudes de la gente en un
concierto.
____________________________________
6. Conocer los antecedentes del SIDA.

____________________________________

7. Clonar un embrin.

____________________________________
62

8. Conocer la opinin de la gente


respecto a un producto nuevo.
____________________________________
9. Encontrar nuevos hallazgos de la
Fsica nuclear.
____________________________________
10. Conocer sobre la
Antigua.

Cultura Egipcia ____________________________________

Ahora realiza 10 ejemplos tomando como base los anteriores.


1.
2.
3.
4.
5.
6.
7.
8.
9.
10.

Actividad 2: Realiza un listado de temas que te gustara investigar.

______________________________________________________________________
______________________________________________________________________
______________________________________________________________________
______________________________________________________________________
______________________________________________________________________
______________________________________________________________________
______________________________________________________________________
______________________________________________________________________
______________________________________________________________________
______________________________________________________________________
______________________________________________________________________
63

______________________________________________________________________
______________________________________________________________________
______________________________________________________________________

Actividad 3: Realiza un mapa mental


investigacin.

64

del tema

Las ramas de la

ESTRUCTURA DEL TRABAJO ACADMICO


La utilizacin del mtodo documental es el ms comn en un trabajo de investigacin,
por ello presentaremos algunas recomendaciones para desarrollar un trabajo de
investigacin documental, los pasos ms importantes son los siguientes:
1. Eleccin del tema.
2. Recopilacin del material.
3. Revisin y registro del material.
4. Delimitacin del tema.
5. Esquema de trabajo.
6. Seleccin del material complementario.
7. Elaboracin de fichas de trabajo.
8. Organizacin de las fichas.
9. Redaccin del borrador.
10. Redaccin final.
Debido a la importancia que cada paso se merece, los trataremos de manera
individual, a continuacin te presentamos cada uno de ellos.
1. Eleccin del tema.
Para elegir el tema de un trabajo es recomendable:
A) El tema debe ser interesante y despertar el inters en los lectores.
B) Cumplir con los objetivos del programa y las orientaciones del profesor.
C) Tomar en cuenta las posibilidades que hay de encontrar informacin accesible
sobre ese tema.
D) Calcular si el tiempo con el que contamos es suficiente para concluir la
investigacin o si ser preferible restringirnos a trabajar sobre un aspecto del
tema. Por ejemplo si el tema general fuera: El general en su laberinto de
Gabriel Garca Mrquez, sera mejor que slo trabajaras sobre: La mujer en el
general en su laberinto de Gabriel Garca Mrquez.

Actividad 4: Selecciona el tema de tu investigacin.


65

2. Recopilacin del material.


Para recopilar el material procura consultar las fuentes, es decir, las obras originales
sobre el tema. Consulta obras actualizadas
como libros, revistas, peridicos,
documentos pblicos o privados, conferencias, etc.
FUENTES DE INFORMACIN
Sistema de Informacin

Fuente Consultable

Biblioteca

Libros

Hemeroteca

Hemerogrficas

Archivo

Peridicos, revistas,
folletos, boletines, etc.
Documentos en general

Audioteca

Material grabado

Audiogrficas

Videoteca

Visual

Videogrficas

Mapoteca

Mapas

Mapogrficas

Museo

Material iconogrfico

Iconogrficas

Centro de Documentacin

Resmenes, notas
bibliogrficas
Informacin directa: datos,
estadsticas
Material de 2 ms
sistemas

Segn la fuente

Centro de Informacin
Centro de Recursos
Mltiples

Tcnicas de Identificacin:
fuente consultada (fichas)
Bibliogrficas

Archivogrfica

Segn la fuente
Segn la fuente

Las fuentes de informacin son instituciones que se encargan de recopilar, guardar,


conservar y difundir material, por ejemplo:
Biblioteca: Libros.
Hemeroteca: Publicaciones impresas peridicamente como: peridico, revistas,
folletos, boletines, etc.
Discoteca o Audioteca: Discos y/o grabaciones (programas de radio, discos,
cassettes, etc.)
Videoteca o Filmoteca: Videograbaciones y/o pelculas cinematogrficas y
programas de televisin.
Mapoteca: Mapas.
Pinacoteca y/o Museos: Pinturas (suelen ser de pintores clebres).
Centro de Informacin: Internet (Documentos, investigaciones, etc. que pueden
ser consultados).
Archivo: Material pblico (ejemplo: documento oficiales e histricos) y/o privado
(ejemplo: informe de actividades de una empresa).

66

Actividad 5: De las fuentes de informacin anteriores, escribe cules son las


que has utilizado.
______________________________________________________________________
______________________________________________________________________
______________________________________________________________________
______________________________________________________________________
______________________________________________________________________
______________________________________________________________________

3. Revisin y registro del material.


Una vez que localizas libros, revistas u otros documentos que te servirn para tu
investigacin, es conveniente que los registres en fichas para localizar con facilidad el
libro en la biblioteca y poder realizar la bibliografa al finalizar tu trabajo.
Los mtodos de registro dependen del tipo de fuente que ests consultando por eso
existen diferentes tipos de fichas que a continuacin se mencionan:
a) Fichas Bibliogrficas.- Recopilan los datos para la identificacin de un libro.
Se realizan en cartoncillo delgado y sus medidas son 12.5 de largo por 7.5
centmetros de ancho.
Para elaborar una ficha bibliogrfica debes localizar los siguientes datos del libro:
Nombre del autor, empezando por el apellido.

Ttulo del libro.

Nmero de edicin, si es la primera no se anota nada.

Pas en el que se public el libro.

Casa editorial que publica el libro.

Fecha de la publicacin.

Nmero de pginas del libro.

67

Ejemplo de ficha bibliogrfica.

Ocampo de Gmez, Aurora M. y Prado Velsquez, Ernesto.


Diccionario de escritores mexicanos
Mxico,
UNAM, Centro de Estudios Literarios,
1998.
pp. 240

b) Fichas hemerogrficas.- Recopilan los datos para la identificacin de un


peridico o revista, con las mismas medidas de las fichas bibliogrficas.
Ejemplo de ficha hemerogrfica.

Roa Bastos en el encuentro


Hacia el fin del milenio.
Peguero, Raquel. La filosofa del lucro, el gran mito de la
edad contempornea: Roa bastos, La jornada, Mxico,
D.F., 3 de septiembre de 19991
pp.31

68

c) Fichas videogrficas.- Recopilan los datos de identificacin de un video.


Ejemplo de ficha videogrfica.

El nombre de la rosa
Jan Jaques Annaud, dir.
Produccin: Bernd Eschinger/Bernd Schaefers
Artistas: Sean Connery, Murria Abraham, Elya Bassin.
120 minutos

d) Fichas de trabajo.- Consignan la informacin importante del texto, datos,


comentarios, etc. De ellas trataremos un tema por separado.

Actividad 6: Elabora una ficha bibliogrfica y una hemerogrfica de los libros


utilizados para la elaboracin de tu trabajo acadmico.

69

Actividad 7: En base a la siguiente narracin, elabora una ficha bibliogrfica.


El da de ayer visit la feria del libro buscando un libro muy especial, un cuento que
para mi representa alegra en mis sobrinos. Un da conoc el cuento en un curso de
lectura y despus se lo cont a mi sobrino, se llama el silln roto; escrito por Alejandro
Mendoza, un hombre con una gran imaginacin, tanta que se gan el premio Filic de
cuento para nios, 1999 y despus lo edit Corunda S. A. de C. V., con la colaboracin
del Consejo Nacional para la Cultura y las Artes. Finalmente lo encontr y me di cuenta
que no es un libro econmico, pero vale la pena tenerlo. Ahora lo leo cada noche a mi
sobrino.
Ficha bibliogrfica

4. Delimitacin del tema.

70

La primera lectura de tu material te permite definir mejor el tema de la investigacin; te


recomendamos trabajar un tema sencillo, es mejor tratar un solo aspecto de un asunto
a fondo, que pretender abarcar un tema extenso de manera superficial. Es conveniente
calcular el tiempo disponible para culminar debidamente el trabajo total.

5. Esquema de trabajo.
Es el esqueleto posible de tu trabajo, las partes o captulos en que podra dividirse.
Como es basado en una primer lectura, puede ser modificado, pero resulta muy til
para que visualices la totalidad del trabajo y la interrelacin de sus partes. Al finalizar
ser el ndice de tu investigacin.
Para ordenar el esquema puedes elegir entre dos sistemas:
Sistema tradicional
Utiliza nmeros romanos, letras
maysculas, nmeros arbigos y letras
minsculas.

Sistema decimal
Se basa en el sistema numrico
decimal, los nmeros se organizan
progresivamente.

Ejemplo del sistema tradicional: el tema es Sometimiento y emancipacin en la mujer


mexicana.
I.

Marco socio-histrico
A. Evolucin de la situacin de la mujer.
1. La edad de piedra
a) Nomadismo
b) Vida comunitaria
c) Matriarcado y patriarcado
d) Sedentarismo.
B. Edad media
C. poca moderna
D. poca contempornea

II.

Marco terico

III.

Mujer mexicana

Presenta ante tu maestro el esquema de trabajo para tu investigacin.


71 en cuenta los comentarios que se
Recuerda que an no es definitivo y toma
presentan en el tema anterior.

6. Seleccin del material complementario.


Una vez delimitado el tema y realizado el esquema de trabajo, te ser ms fcil
discriminar entre el material que conseguiste de forma inicial, y buscar adems otras
fuentes en libros, artculos, conferencias, que estn directamente relacionadas con tu
esquema y complementen mejor tu acervo. Toma en cuenta registrar los nuevos datos
bibliogrficos.

7. Elaboracin de fichas de trabajo.


Cuando consultas un libro para realizar una investigacin, lo primero que debes hacer
es elaborar una ficha bibliogrfica, pero una vez que empiezas a leerlo, tienes que ir
tomando notas que te servirn para fundamentar tu trabajo y estas notas se llaman
fichas de trabajo.
Las fichas de trabajo incluyen notas, ideas, etc., son muy tiles para recordar datos,
organizar tu redaccin, anotar la bibliografa y conservar la memoria del material
consultado. Se realizan en cartoncillo delgado y sus medidas son 20 centmetros de
largo por 12 de ancho.

Las fichas de trabajo se clasifican en:


Resumen
Parfrasis
Sntesis
Cuadro sinptico
Mapa conceptual
De comentario
Cita textual

72

Para facilitar tu comprensin de la estructura de una ficha de trabajo te presentamos el


siguiente esquema donde se muestran las partes de la ficha.

1/5
Tema que desarrolla la ficha
Autor
Ttulo del libro
El cuerpo de la ficha presenta una de las formas en las
cuales se puede realizar como el resumen, parfrasis,
sntesis, cuadro sinptico, mapa conceptual, comentario o
cita textual.

Observa el siguiente ejemplo de una ficha de cita textual:

Las Bibliotecas
Paredes Cavara, Elia
Prontuario de lectura
La biblioteca, es el lugar donde se almacenan los libros. En
efecto en una biblioteca se organiza, conservan, y divulgan
los libros que el conocimiento humano ha creado, por lo que
es una institucin indispensable para la vida acadmica.

73

Actividad 8: De los textos que has revisado, realiza segn la informacin que
tengas tres fichas, ya sea de cita textual, de parfrasis, de resumen, etc.

8. Organizacin de las fichas.


Separa las fichas que consideres repetidas o irrelevantes. Es conveniente que anotes
en tus fichas de trabajo el captulo de tu esquema con el cual tienen relacin, las
enumeres segn el orden coherente en que las podras utilizar en el momento de
redactar.
74

9. Redaccin del borrador.


El borrador constituye tu primer intento de poder organizar la redaccin de tus ideas
enriquecidas con las aportaciones de tus fichas de trabajo. Si no tienes mucha
experiencia, escribe tus ideas tal como fluyan en tu mente y trata de ordenarlas en
forma progresiva, corrige constantemente, eliminando las palabras que no son
importantes o son repetitivas.
Tomando en cuenta que las cualidades fundamentales de la redaccin son
claridad, sencillez, concisin, sinceridad, originalidad, y naturalidad, en el momento
de redactar no utilices palabras rebuscadas, usa un diccionario de sinnimos y
antnimos, en general checa muy bien tu narracin. Antes de entregar cualquier
trabajo revisa los siguientes aspectos:
a. Mrgenes.
b. Sangras.
c. Limpieza.
d. Legibilidad.
e. Ortografa.
f. Puntuacin.
g. Distribucin de las ideas en prrafos.
h. Estructura del texto.
i. Mensaje coherente, efectivo y original.
j. Vocabulario acertado.

Recuerda que tu trabajo debe ser original, constantemente revisa,


vuelve a leer.

corrige y

10. Redaccin final.


Al redactar tu trabajo debes considerar todo lo anterior que hemos tratado, pues el
contenido es muy importante, tanto como la presentacin de lo cual hablaremos en el
siguiente apartado.

Actividad 9: Elabora un cuadro sinptico del tema: Estructura del trabajo


acadmico.
75

PRESENTACIN DEL TRABAJO ACADMICO


Aunque lo ms importante en la redaccin es el contenido y la calidad de las ideas
expuestas, al pasar en limpio tu trabajo debes cuidar la presentacin y distribucin
adecuadas de todas las partes de tu trabajo.
76

Si hablamos de la estructura general de un trabajo acadmico, se pede estructurar de la


siguiente manera:
a) Portada: Incluye todos los datos mnimos de identificacin: nombre de la escuela,
ttulo del trabajo, ttulo del curso o materia, nombre del profesor, nombre del alumno,
grupo, lugar y ao en el que se presenta. Veamos el ejemplo.

UNIVERSIDAD AUTNOMA DEL ESTADO DE HIDALGO


Escuela Preparatoria Ricardo Flores Magn

LA MUJER EN:
EL GENERAL EN SU LABERINTO
DE GABRIEL GARCA MRQUEZ
LITERATURA
Profesora: ngeles Rodrguez Iglesias

Julio Garca Rivas


Grupo: 3 Turno: Matutino

Pachuca, Hidalgo a 5 de abril del 2005

b)
Prlogo o Introduccin: El alumno elige cualquiera de estas formas para iniciar
su trabajo. Cada una de ellas tiene su propia forma, el prlogo o prefacio es un texto
del mismo autor en el que explica de modo breve los motivos que lo llevaron a escribir
el tema, el proceso que sigui para elaborarlo, las dificultades que enfrent, la utilidad
de su aportacin y su agradecimiento a quienes facilitaron su tarea. Puede ser escrito
por otra persona para alabar la obra presentada por el autor.
77

La introduccin, como su nombre lo indica, introduce al lector al contenido de la obra,


lo orienta acerca de sus temas, su organizacin y secuencia; se pueden agregar
antecedentes que faciliten la comprensin.
c)
ndice: Es el indicador del nmero de pgina donde se desarrolla un tema en
especfico.
d)
Texto (es el cuerpo del trabajo): Es la parte ms importante de todo el trabajo.
El texto o cuerpo debe seguir una secuencia lgica, ser claro, coherente y ordenado
desde su estructuracin total, hasta la redaccin de cada uno de sus enunciados y
prrafos.
e)
Conclusiones: Las conclusiones pueden enumerarse progresivamente despus
del contenido del trabajo. Deben ser breves, pues slo sintetizan los aspectos ms
importantes de la investigacin, las reflexiones y las posibles sugerencias que pudieran
haberse generado al desarrollar el trabajo.
f)
Aparato Crtico (citas y bibliografa): Las citas pueden ser referencias
bibliogrficas de las ideas expuestas por otro autor o textos consultados. La bibliografa
es la lista de los datos bibliogrficos de todos los textos consultados.
Una forma moderna de marcar las referencias, es la llamada de autor-fecha que se
colocan entre parntesis inmediatamente despus de la cita. Sus partes son parntesis,
apellido del autor, coma, ao de publicacin, coma, pgina de referencia y parntesis;
veamos el ejemplo:
Es difcil decir si la diferencia en las actitudes de los adolescentes en su respuesta
sexual se basa fundamentalmente en la biologa o en la cultura. (Papalia, 1990, pp.
431)

En la bibliografa este texto aparecera as:


Papalia, D. (1990). Desarrollo Humano.
Mxico, McGraw-Hill.

INFORME

78

El informe es uno de los documentos utilizados con mayor frecuencia en las empresas,
en las instituciones pblicas y privadas, en la investigacin, y en el medio acadmico.
Tiene la finalidad de presentar hechos o investigaciones realizadas con un propsito
especfico: presentacin de informacin, anlisis o exposicin de hechos, resumen de
actividades, previsin de situaciones, solucin de problemas y recomendaciones, entre
otros. Los asuntos tratados en los informes pueden referirse a problemas tcnicos,
administrativos, financieros, acadmicos, etc.
CLASIFICACIN
Existen diferentes criterios para la clasificacin de los informes.
En cuanto al contenido y al propsito del informe se pueden identificar tres tipos:
1.- De trabajo
2.- Tcnico
3.- Acadmico
ESTRUCTUR A DEL INFORME
Portada: Es la primera pgina del informe, contiene los siguientes puntos:
-

Nombre de la empresa o institucin que realiz la investigacin. Se puede utilizar


el membrete.
Ttulo del informe. En caso de lo que requiera, se incluye el periodo que cubren
las actividades referidas.
Nombre del destinatario del informe, incluido el cargo.
Nombre del responsable de la investigacin.
Lugar y fecha.

ndice: Se presenta, de manera analtica, el contenido total del informe, indicando el


nmero de la pgina donde se encuentra.
Objetivo: Describe la finalidad del informe, de manera tal que el directivo conozca
con claridad de lo que trata el informe. Su redaccin inicia con un verbo en infinitivo
(ar, er, ir).
Problema: Descripcin del problema en forma resumida.
Anlisis o investigacin: Relata en forma pormenorizada y resumida el anlisis o la
investigacin realizada.
Sugerencias o recomendaciones .
Conclusiones: El planteamiento de las causas o efectos del problema y sus posibles
soluciones.

79

Anexos: Son agregados que incluyen informacin complementaria acerca del


desarrollo de la investigacin: tablas, grficas, cuadros, cuestionarios, etc.
Glosario: Definicin o explicacin de trminos especficos. Tiene la finalidad de facilitar
la comprensin del informe al lector.
Referencias y bibliografa: fuentes de la informacin
Datos del autor: Nombre o cargo, de acuerdo con la informacin que aparezca en la
portada.
Fecha: La fecha cierra el documento.
a) Informe de Trabajo
El informe de trabajo es un documento de carcter interno, de uso frecuente en
empresas e instituciones pblicas y privadas.
El propsito del informe de trabajo es comunicar al lector de manera objetiva y
sistematizada los avances de un proyecto o el funcionamiento de algn departamento
de la empresa o institucin, adems de evaluar el cumplimiento de un programa
establecido previamente.
En el informe de trabajo se consideran los siguientes aspectos:
-

Describe una situacin o un problema.


Incluye recomendaciones o sugiere posibles soluciones.
Puede complementarse con otros documentos: dibujos, planos, fotografas,
tablas estadsticas, etc.

b) Informe Tcnico
El informe tcnico est directamente vinculado con los trabajos de investigacin
cientfica y tecnolgica. Es el principal instrumento para la difusin e intercambio de
nuevos conocimientos y aportaciones relevantes, entre la comunidad dedicada a la
investigacin en las diversas reas de la ciencia y la tecnologa.
Estructura del Informe Tcnico:
Introduccin: De manera objetiva se presentan los antecedentes y la fundamentacin
del trabajo. Se debe enfatizar el inters o la importancia del mismo.
Resumen: Es una breve recapitulacin del contenido total del trabajo. Debe permitir a
los lectores identificar con rapidez el contenido bsico del informe y determinar su
importancia.

80

Desarrollo: Es la parte medular y se le considera como la correspondiente a


materiales, mtodos y resultados. Es recomendable dividirlo en captulos que, a su vez
deben constituir una unidad orgnica.
Discusin y conclusiones: El primer punto corresponde a una interpretacin objetiva
e imparcial del autor con respecto a los resultados obtenidos. El segundo es la
integracin de los puntos esenciales del trabajo.
Apndices o anexos: Es el material que complementa el desarrollo del tema (cuadros,
tablas, etc.).
Referencias bibliogrficas: Su importancia reside no slo en dar crdito a los autores
en cuyos trabajos se basa una investigacin, sino tambin en ofrecer a lector la
posicin de consultar directamente las fuentes.
c) Informe acadmico
El informe acadmico tiene como principal propsito presentar los resultados de una
investigacin realizada en el proceso de enseanza- aprendizaje.
Para la elaboracin de un informe acadmico se requiere de manera sistemtica llevar
a cabo un proceso de investigacin documental. La investigacin consiste bsicamente
en una bsqueda, seleccin y organizacin de informacin y la presentacin del trabajo
por escrito (informe acadmico).
Procedimiento:
- Consulta de las fuentes de informacin.
- Seleccin de la bibliografa.
- Acopio de la informacin en fichas de trabajo.
- Organizacin de las fichas.
- Elaboracin de un esquema del trabajo (modelo del escarabajo o indagatorio).
- Redaccin del borrador.
- Redaccin final.

En cuanto a la organizacin general del informe acadmico, se puede estructurar de la


siguiente manera:
1.
2.
3.
4.
5.
6.

Portada
ndice
Introduccin
Desarrollo o cuerpo del trabajo
Conclusiones
Bibliografa
81

7. Apndice o anexos (opcional)

Actividad 10: En equipo, redacta un informe de trabajo sobre el funcionamiento de


alguno de los servicios educativos que ofrece la escuela. Debe comprender las
actividades realizadas en un periodo determinado (extensin mxima cinco
cuartillas).

E N SAYO

El ensayo es un escrito, en el cual el autor expone sus ideas y reflexiones personales


relativas a un tema determinado, es decir, se trata de un trabajo de opinin.
El trmino ensayo proviene del latn: exagium, cuyo significado se refiere al acto de
pensar algo.
El aspecto fundamental del ensayo es el punto de vista personal y analtico que asume
el autor al abordar un tema determinado de manera intensa y metdica. ste escrito no
se propone definir verdades definitivas, sino remover la inteligencia e inquietar los
espritus de los lectores. Su objetivo es hacer pensar.
82

CAR AC TERSTIC AS:


En cuanto a sus caractersticas externas, se indican las siguientes:

Por lo general, el ensayo se escribe en hojas sueltas, impresas en computadora,


con el fin de publicarlo luego en algn medio impreso (un libro o una revista).
En cuanto a su extensin, los ensayos suelen ser escritos breves.
La estructura u organizacin de un ensayo generalmente es la siguiente: Un
planteamiento introductorio, un desarrollo del argumento o de las ideas y una
conclusin.

En cuanto a sus caractersticas internas, se indican las siguientes:


Los ensayos pertenecen al prototipo de textos expositivos y explicativos.
Contenido: En ellos se presentan las ideas personales del autor, sobre un tema o
problema determinado.
En los ensayos se utiliza un lenguaje formal, aunque no tan tcnico o acadmico.
Se puede utilizar un lenguaje literario o potico.

En Mxico, los escritores que se han dedicado ms al ensayo son: Jaime Torres Bodet,
Octavio Paz y Alfonso Reyes.
ORIENTACIONES PRCTIC AS

El primer paso es seleccionar el tema o el problema que te interese y hasta que te


apasione, sobre el cual elaborar un ensayo.

Buscar fuentes de informacin en las que se hable de ese tema. Aunque en el


ensayo expresars tu opinin personal, necesitas saber lo que han dicho otras
personas, con el fin de que no repitas cosas que ya dijeron otros.

Al leer las fuentes de informacin que encuentres, elabora las correspondientes


fichas de apoyo: bibliogrficas, hemerogrficas, de registro, de pgina electrnica, de
trabajo, etctera.

Conforme leas y elabores tus fichas, define tu propia posicin con relacin a ese
tema o problema, ya que en funcin de ella desarrollars tu escrito.

83

Esquema General del


Ensayo:
Portada
ndice
Presentacin
Desarrollo del tema
(Introdu-ccin,
Desarrollo y
Conclusin)
Referencias
Bibliogrficas

Actividad 11: Lee atentamente el siguiente ensayo sobre Don Juan Tenorio de Zorrilla
y contesta las interrogantes de la lectura.

Ramiro de Maetzu
Don Juan es humano, vienen a decir los enemigos que recientemente le han salido al
ms universal de los fantasmas literarios. A los que se ha de contestar con una
pregunta: No consistir precisamente su grandeza en que no es humano en la medida
en que lo son los mitos? Lo engendr la fantasa hispnica, pero no la realidad
espaola; surgi de la leyenda, no de la historia; lo produjo la imaginacin creadora, no
la observacin. Basta ponerse a mirar la realidad con ojos francos y, sobre todo con
ojos modernos, para que se muestre la inconciencia de Don Juan. Qu escndalo es
este de un seor que se dedica a conquistar mujeres y no se enamora de ninguna?
Cmo puede tolerarse que no crea en la retrica del amor, su principal figura? Si Don
Juan fuera humano, se enamorara ms o menos de cada una de las mujeres a las que
persigue, por que no las perseguira si no las codiciase, y no las codiciara si no
84

estuviese enamorado de ellas. Pero un Don Juan que se enamora no es ya un Don


Juan...
El secreto de la fascinacin que Don Juan ejerce consiste precisamente en su energa
inagotable. Esa infinitud depender de que no se enamora, de que no gasta o de que
es intrnsecamente inagotable. No lo sabemos, ni es necesario averiguarlo. Alguna vez
me he figurado que el enigma de Don Juan se descifra imaginndole alguna secreta
rabia a las mujeres, como la que debi de sentir Don Ramiro de Rodrguez Larreta, al
averiguar que su padre era morisco y no un altivo caballero castellano de quien
imaginaba descender. Pero esta hiptesis de un alma amargada, que sirve
perfectamente en el drama de Zorrilla, para explicarnos, en los actos ltimos el reto
sacrlego de los muertos para que asistan a su banquete, no se acuerda, en cambio,
con la jovialidad y el mpetu con que se nos presenta el personaje y que ntegro se
conserva hasta que pierde a Doa Ins.
1. Cul es el tema del que habla el ensayo?
______________________________________________________________________
______________________________________________________________________
______________________________________________________________________
2. Cul era la postura de Ramiro de Maetzu, est a favor o en contra del
personaje de Don Juan?
______________________________________________________________________
______________________________________________________________________
______________________________________________________________________
3. Cul es tu comentario sobre este ensayo?
______________________________________________________________________
______________________________________________________________________
______________________________________________________________________
4. Qu ttulo le pondras?
______________________________________________________________________
______________________________________________________________________
______________________________________________________________________

Actividad 12: Redacta un ensayo del tema


tomando en cuenta los siguientes puntos:

Introduccin

"El Hbito de la Lectura

Ideas principales
1. Por qu es importante la lectura?
2. Por qu es importante comprender lo que se lee?
85

Desarrollo

1. Forma adecuada de hacer una lectura comprensiva.


2. Maneras de incrementar el hbito de la lectura.
3. Lectura de obras literarias.
4. Las fuentes de lectura disponibles para los alumnos
(peridicos, revistas, boletines, libros de texto).
5. Bibliotecas vecinas a la escuela.

Conclusin

1. Beneficios que obtenemos por medio de la lectura


(explicarlos).

Despus de escribir un ensayo, podrs evaluarlo contestando las preguntas siguientes:


1.- Cuntas cuartillas se usaron para redactar el ensayo?
2. El ensayo es breve?
3.- Qu nos ensea el tema desarrollado?
4.- Contiene tu opinin personal con respecto al asunto tratado?
5.- El ensayo est escrito en prosa?

MONOGRAFA

La intencin comunicativa de la Monografa es transmitir a otras personas informacin


relacionada con eventos dignos de recordar, as como tambin cumplen con una
funcin referencial o informativa utilizando un lenguaje cientfico.

Monografa:

Proviene del griego monos: un solo, y graf: escritura.


Significa escrito sobre un solo tema de algo preciso.

86

Es una descripcin y tratado especial de un solo asunto o de cierta parte de una


ciencia o arte.

Es un trmino muy general y abarca cualquier escrito que trate un solo tema.

Para hacer una monografa se requiere un trabajo de investigacin


documental. Por eso las monografas se estructuran de la siguiente
manera: Portada, ndice, introduccin, desarrollo (con uno o varios
captulos), conclusiones y bibliografa.

Caractersticas de una Monografa:


1. Caractersticas externas:
Son el resultado de trabajos de investigacin destinadas a publicarse para
que otros las lean.
Estos trabajos o estudios se presentan a manera de libros.
Se presentan impresas en computadora.

La estructura y organizacin de una monografa cuando es de carcter


histrico se redacta en orden cronolgico.
Su extensin es variable, dependiendo del tema que se est tratando
(puede haber de 10 a 15 pginas hasta 300 400 pginas).
2. Caractersticas internas:
El asunto tema siempre es muy particularizado.
Debido a su carcter cientfico utilizan un lenguaje formal, tcnico y
acadmico.

87

Paso para elaborar una monografa:


1. Definir el tema: Asunto sobre que tratar la
monografa.
2. Buscar informacin: Libros, revistas,
peridicos, internet, entre otros.
3. Leer: Dar lectura a todo el material y hacer
fichas bibliogrficas, hemerogrficas, de
registro de pgina electrnica y de trabajo.
4. Redactar: El contenido, incluyendo el
desarrollo, los anexos y las referencias
bibliogrficas.
5. Elaborar: la introduccin, el ndice y la
portada.

Con el fin de ilustrar su contenido, las monografas pueden contener dibujos o


fotografas acerca del tema que se aborda. As mismo pueden incluir anexos sobre
elementos que no sean sustanciales para la presentacin, pero que sean de inters
general.

Actividad 13: Realiza una monografa, seleccionando previamente el tema y


atendiendo los requisitos vistos en clase.

88

AUTOEVALUACIN
Instrucciones: Lee detenidamente cada cuestionamiento y subraya la respuesta
correcta.
1.- Para conocer datos sobre la vida y obra de Don Ignacio Ramrez El Nigromante
(1818-1879), un maestro pidi a sus alumnos realizar una investigacin.
Cul es el tipo de investigacin ms apropiada a utilizar?
A) De campo.
B) Documental.
C) De laboratorio.
D) Observacin.
2.- Un maestro entrega a sus alumnos estadsticas sobre poblacin con los siguientes
rubros: vivienda, servicios e ingresos. Les pide que realicen una investigacin de campo
en una colonia cercana para corroborar esos datos.
Qu deben hacer?
A) Conseguir una grabadora para entrevistas.
B) Consultar el anuario del INEGI.
C) Procesar muestras de suelo en un laboratorio.
D) Llamar por telfono a la oficina de estadstica para ampliar la informacin.
3.- Un maestro informa a sus alumnos que el virus de la gripe Aviar fue detectado en un
gato en Alemania, por lo que se estn tomando medidas en el caso. Qu mensaje
quiere transmitir el profesor?
89

A) Informar que hay varios tipos de gripe.


B) Que lean las noticias de la prensa.
C) Que se interesen por la investigacin experimental o de laboratorio.
D) Que extremen precauciones en el consumo de Aves.
4.- Un alumno va a entregar una investigacin sobre la vida y obra de Pedro Caldern
de la Barca. Cmo debe citar la siguiente frase?
A) ...todo es segn el color del cristal con que se mira.
B) Todo es segn el color del cristal con que se mira (De la Barca: 1991).
C) Dice Caldern que los anteojos cambian el paisaje.
D) Dice el autor que la vida es un sueo.

5.- Una investigacin exige el seguimiento de un proceso. En qu parte del proceso


son tiles el uso de preguntas como: qu?, dnde?, cundo?, cmo?, y otras
similares?
A) Delimitacin del tema.
B) Redaccin del borrador.
C) Esquema de trabajo.
D) Eleccin del tema.
6.- Los elementos de una ficha textual son: 1. Texto de la cita; 2.Tema; 3. Obra
consultada; 4. Fecha; 5. Autor. Si las ordenamos en una tarjeta de trabajo, de arriba
hacia abajo cul sera el orden correcto?
A) 5,4,3,1,2
B) 1,5,3,2,4
C) 3,4,2,5,1
D) 4,2,3,5,1
7.- Los siguientes elementos corresponden a uno de los documentos que analizamos
en clase. Observa los elementos que proporcionamos e identifica el tipo de documento
a que corresponde.
Objetivo, Problema, Anlisis o Investigacin y Sugerencias o Recomendaciones.
A) Ensayo.
B) Resea.
C) Monografa.
D) Informe.

90

8.- El siguiente texto corresponde a uno de los elementos del informe. Identifcalo: El
13% de los alumnos carece de estrategias y hbitos de estudio.
A) Glosario.
B) Objetivo.
C) Conclusiones.
D) Anexo.
9.- Un alumno del CECyTE realiz una investigacin y consult un libro de
investigacin. Cul es la forma correcta en que debi registrarlo?
A) J. W. Best, Como investigar en educacin, Ed. Morata, 3. reimpr., 1982.
B) Best, J. W. (1982). Como investigar en educacin. Mxico: Ed. Morata.
C) Como investigar en educacin. J. W. Best. Mxico 1982.
D) Best J. W., ediciones Ed. Morata, Como investigar en educacin.

Lee el siguiente texto y contesta las preguntas 10, 11 y 12.


El Principito es una inagotable y profunda obra potica. En este libro Saint-Exupry
hace muchas sugerencias de las que se pueden obtener sustanciales conclusiones.
Posicionado en un diminuto asteroide, localizado a miles de kilmetros de la tierra, donde se
libran cruentas batallas para sobrevivir, un angelical nio intenta rescatar las mayores virtudes
del ser humano dejadas en el olvido. Esta obra es un verdadero testimonio de amor y ternura
inagotable.

10.- A qu tipo de escrito pertenece el texto?


A) Ensayo.
B) Resea.
C) Monografa.
D) Informe.
11.- El texto anterior nos permite saber qu tipo de lenguaje utiliza el autor, que la obra
est escrita en prosa y como maneja los conceptos. Esto significa que podemos
determinar acerca del texto su
A) Estructura.
B) Estilo.
C) Temtica.
D) Mensaje.
12.- El hecho de que en el texto se mencione el nombre de la obra y sus aspectos
relevantes, nos permite conocer su
A) Estructura.
B) Estilo.
91

C) Temtica.
D) Mensaje.

Lee el siguiente prrafo del libro Don Juan Tenorio del espaol Jos Zorrilla (1817
1893) y contesta las preguntas 13 y 14.
Don Juan es humano, vienen a decir los enemigos que recientemente le han salido al ms
universal de los fantasmas literarios. A los que se ha de contestar con una pregunta: No consistir
precisamente su grandeza en que no es humano en la medida en que lo son los mitos? Lo engendr la
fantasa hispnica, pero no la realidad espaola; surgi de la leyenda, no de la historia; lo produjo la
imaginacin creadora, no la observacin. Basta ponerse a mirar la realidad con ojos francos y, sobre
todo con ojos modernos, para que se muestre la inconciencia de Don Juan. Qu escndalo es este de
un seor que se dedica a conquistar mujeres y no se enamora de ninguna? Cmo puede tolerarse que
no crea en la retrica del amor, su principal figura? Si Don Juan fuera humano, se enamorara ms o
menos de cada una de las mujeres a las que persigue, por que no las perseguira si no las codiciase, y
no las codiciara si no estuviese enamorado de ellas. Pero un Don Juan que se enamora no es ya un Don
Juan... (Ramiro de Maetzu)

13.- A qu clase de escrito pertenece el texto anterior?


A) Ensayo.
B) Resea.
C) Monografa.
D) Informe.
14.- Cul es la postura del autor del texto (Ramiro de Maetzu) respecto a Don Juan
Tenorio?
A) A favor.
B) Neutral.
C) En Contra.
D) Discriminatoria.
15.- Ramiro entreg un ensayo al maestro de LEXO que contiene los siguientes
elementos: Portada, ndice y desarrollo del tema, cul le falt?
A) El Problema.
B) Los Objetivos.
C) El ttulo del trabajo.
D) Las Referencias bibliogrficas.
16.- En un ensayo el desarrollo del tema puede contener varios elementos. Cul de
las opciones los contiene a todos?
A) Desarrollo de las ideas, objetivos y presentacin.
B) Planteamiento introductorio, apndice y portada.
C) Conclusiones, ndice y bibliografa.
D) Planteamiento introductorio, desarrollo de las ideas y conclusiones.
92

17.- Un ensayo tiene caractersticas internas y externas, enseguida 3 opciones


contienen caractersticas de ambos tipos y solo una contiene exclusivamente
caractersticas internas. Identifcala:
A) Texto expositivo, ideas personales del autor, lenguaje formal.
B) Ideas personales, escrito en hojas sueltas, escrito breve.
C) Textos expositivos, planteamiento introductorio, desarrollo del argumento.
D) Conclusiones, lenguaje formal, impreso en computadora.
18.- Al realizar una monografa sobre Los inventos de 1980-2000 un estudiante
ordena los temas de la siguiente manera: 1970 Fibra ptica, 1971 E-mail, 1971
Microprocesador, 1971 Calculadora de bolsillo, 1978 Lector lser de disco, 1979
Walkman, 1979 Telfono celular, 1982 CD, 1983 Protocolo de Internet, 1985 Huella
ADN, 1989 World Wide Web, 1995 DVD.
A qu tipo de ordenamiento corresponde?
A) Cronolgico.
B) Alfabtico.
C) Temtico.
D) Por pas.
19.- Para realizar una monografa se requiere llevar a cabo una serie de pasos. En las
opciones siguientes solo una los contiene en el orden correcto. Identifcala:
A) Buscar informacin, Definir el problema, leer, redactar y elaborar el trabajo.
B) Leer, definir el problema, buscar informacin, redactar y elaborar el trabajo.
C) Definir el problema, buscar informacin, leer, redactar y elaborar el trabajo.
D) Buscar informacin, definir el problema, leer, redactar y elaborar el trabajo.
20.- Si un maestro encarga a sus discpulos un reporte referido a su municipio,
consignando los aspectos: geogrfico, econmico, poltico e histrico.
Qu tipo de reporte est solicitando?
A) Informe.
B) Monografa.
C) Ensayo.
D) Resea.

93

U
N
I
D
A
D
94

Expresin Oral

Hablar ante el
pblico con el
propsito de
convencerlo o
persuadirlo,
sin duda,
es un reto para
muchas personas,
pues para lograrlo
se requiere
inteligencia,
experiencia y
fuerza de voluntad.

95

EXPRESIN
ORAL

La comunicacin oral desempea un papel primordial en el establecimiento de las


relaciones interpersonales, tanto en el mbito social como en el profesional.
La capacidad de expresarse oralmente es una de las aptitudes que ms se valora y se
exige para el reconocimiento profesional de una persona. Para lograr expresarse con
propiedad se requieren conocimientos y desarrollo de ciertas habilidades que pueden
adquirirse mediante una prctica constante.
Para cualquier persona la capacidad de expresarse con propiedad, ya sea en un
dilogo o ante un auditorio, constituye un requisito indispensable. Cuando una persona
comunica sus necesidades e ideas a otros para lograr influir en la conciencia de los
dems, debe hablar bien, de modo coherente, convincente y preciso. Es decir, la
comprensin, la persuasin y an el entretenimiento que se logren transmitir y compartir
a travs del mensaje requieren de una comunicacin eficaz.
La comunicacin oral comparte elementos con la comunicacin escrita en cuanto a las
cualidades o requisitos, como la claridad, la precisin, el adecuado uso del lenguaje, el
orden, etc., que contribuyen a hacer ms eficaz la comunicacin. La comunicacin oral,
en la mayora de los casos, requiere de una preparacin, es decir, la bsqueda de la
informacin mediante la lectura y de la organizacin del material escrito.

96

La expresin oral
consiste en el empleo
de la palabra hablada
y del pensamiento,
en forma correcta,
elegante y clara.

PRINCIPIOS BSICOS PARA HABLAR EFICAZMENTE

ADQUISICIN DE LOS CONOCIMIENTOS BSICOS


Siempre o casi siempre, el acto de la comunicacin surge como
una necesidad de transmitir o compartir ideas, conocimientos,
sentimientos, sensaciones, necesidades, etc. Si no se tiene la
suficiente claridad sobre lo que se quiere decir, o simplemente no
se cuenta con los conocimientos indispensables, difcilmente se
podr cumplir con el propsito de la comunicacin.

En la medida en que una persona conoce el tema del cual hablar, tendr mayor
facilidad para comunicarlo y adems tendr ms seguridad en si mismo.

AUTODOMINIO

El autodominio se ha definido como la cumbre del logro humano, es semejante a una


semilla, ya que el ser humano puede cultivarla, es decir, sembrarla, regarla, abonarla y
cosecharla.
Manifiestas autodominio cuando:
Mantienes tus cualidades fsicas (volumen, velocidad, tono, ademanes y arreglo
personal) bajo control. Por ejemplo, cuando usas un tono de voz agradable al decirle:
"Comprendo su preocupacin y punto de vista. Permtame explicarle mis razones y
comprender por qu lo dije", en vez de hablarle speramente, en tono brusco.
Mantienes tus enfoques y puntos de vista (informacin, impacto, contacto y
entretenimiento) bajo control. Por ejemplo, cuando dices: "Algunas personas tienden a
97

ver con ojos muy crticos a los de otra raza o posicin social", en vez de decir: "Todos
somos unos racistas y clasistas irreformables". Eso significa tratar al pblico con
respeto. O cuando res con el auditorio a raz de un suceso inocente que les hizo gracia
a todos, en vez de ponerles mala cara.
Mantienes tu tiempo (duracin del discurso y las diferentes secciones del mismo y tu
puntualidad) bajo control. Por ejemplo, dices: "Dispongo de 10 minutos para responder
sus preguntas", "espero que comprendan que, aunque me gustara explayarme ms, es
importante mantener mi exposicin dentro del tiempo asignado". O cuando una persona
llega tarde porque tuvo un suceso imprevisto que estuvo ms all de su control,
mostrndole comprensin por sus limitaciones.
S, una de las mejores "inversiones" que puedes hacer es, sin duda, aprender a
exponer tus ideas con autodominio. Porque solo as sentirs que tu exposicin
verdaderamente adquiere una potencia inusitada y contribuirs al enriquecimiento de la
inteligencia y nobleza de tu auditorio.
Cuando seas parte del auditorio y la persona que expone o alguien del auditorio pierda
el control (levantando la voz o hablando tonteras), ten presente que perder el equilibrio
o la compostura es fcil. Recuerda que solo unos pocos alcanzan la cumbre del logro
humano: El autodominio. La mayora ignora su existencia. Por eso, aunque patines de
vez en cuando diciendo una tontera o falacia involuntaria, lucha por mantenerte bajo
control, en equilibrio. Entonces, hasta en dosis pequeas, tu poder ser tremendo. Tu
palabra dar en el clavo ms a menudo y contribuirs al desarrollo de todos.

Las personas se quejan de ciertas reacciones fsicas o emocionales


antes de hablar en pblico: les tiemblan o sudan las manos, respiran
ms rpido, se sienten dbiles, mareadas o tensas, se sienten
CAN
ALIZAR
EL miedo
NERVIOSISMO
inseguras,
tienen
a equivocarse, sienten ganas de rer, etc.
Frecuentemente el individuo duda de su capacidad de enfrentar el reto
de hablar ante un pblico. Cree que tal sensacin, tal ansiedad, no la
sufre nadie como l. El nerviosismo que sentimos al pensar en hablar
ante un grupo es una respuesta emocional y fsica normal, la cual nos
indica que estamos enfrentndonos a una situacin importante que tal
vez resulte peligrosa.
98

El hombre es un ser social que requiere ser aceptado y respetado. Al exponerse para
ser enjuiciado y/o evaluado por los dems, se arriesga a no lograr la aceptacin social.
Ante la situacin de hablar en pblico y cualquiera otra que la persona considera
importante y en la que lleva el riesgo de no lograr la aceptacin social, la naturaleza le
proporciona una energa de emergencia, conocida como adrenalina, para ayudar a
enfrentar la situacin eficazmente.

Nuestras reacciones fsicas y psicolgicas, ante cualquier situacin


importante para nosotros, son normales. Por tanto, no es malo sentirse
nervioso antes de hablar en pblico. No somos anormales, ni cobardes
si experimentamos tales reacciones. Por el contrario, somos normales
porque valoramos la aprobacin social y nos esforzamos para lograrlo.

Recordemos que el nerviosismo es la energa de emergencia que


nos da la naturaleza para superar tales situaciones. Lo que hay
que hacer es aprender a utilizar esa energa de manera correcta
para lograr la aprobacin del pblico.

99

Esquema
Cultivar la actitud
receptiva

Practicar

Canalizar
el
nerviosismo

Informarse

Relajarse

Prepararse para
preguntas

Comprometerse
con las ideas

COMPROMETERSE CON LAS IDEAS


Quien cree estar imposibilitado para hablar en pblico o rehuye
enfrentarse con la situacin de comunicacin oral est demasiado
centrado en s mismo. Le preocupa como se ve ante el pblico, su
apariencia fsica, su vestimenta, etc. Las personas que se preocupan
demasiado en como los percibe el pblico no pueden concentrarse
en el mensaje que presentan. Su energa y atencin estn centradas
en su persona.
Si logramos comprometernos con nuestras ideas, convencernos de que tenemos un
mensaje importante que queremos compartir con el pblico, desviaremos la atencin
que mantenemos en nosotros mismos hacia el mensaje que presentamos. De esta
forma, dedicaramos nuestra energa y atencin al mensaje y evitaramos ponernos
nerviosos. Nuestro pensamiento estara dirigido a un objeto: al mensaje y a nuestra
tarea de impartirlo con el pblico. No podemos estar nerviosos y al mismo tiempo
concentrarnos en nuestro mensaje y la tarea que conlleva.
Para comprometerse con las ideas se deben considerar las limitaciones que imponen la
situacin y el pblico. Si somos maduros, si se reflexiona sobre las ideas, sabremos
que se puede lograr dentro de estas limitaciones.

INFORMARSE

100

A veces, el miedo se manifiesta, cuando la persona se enfrenta ante un pblico, como


una preocupacin en su intento por lograr que su personalidad sea aceptada y que el
pblico considere interesante el tema que presenta.

Un orador interesante es una persona interesante. Una persona


interesante se mantiene informada de los asuntos de importancia pblica;
no se satisface con leer el peridico local, que le repite las noticias de las
agencias internacionales en forma tan espordica que no se prestan a la
interrelacin de interpretacin de los hechos; tampoco se limita a
escuchar las informaciones de los noticieros de televisin, que tambin
presentan las noticias de la misma manera espordica y superficial.

La persona educada busca informacin en


fuentes de alta credibilidad, con frecuencia lee
revistas semanales, quincenales o mensuales;
sabe quienes
son los periodistasinvestigadores que escriben para tales fuentes
de alta credibilidad; conoce la especialidad
acadmica de los periodistas o articulistas que
suele consultar: si tienen formacin de
economista, historiador, cientfico, poltico,
bilogo, etc., y los consulta como un modo de
prepararse para interpretar los hechos.

PRACTICAR LA COMUNICACIN EN PBLICO


La prctica es la recomendacin que con mayor frecuencia dan los
expertos para reducir el miedo. Con la experiencia se obtiene
confianza. La recomendacin entonces es que aceptes cualquier
oportunidad de hablar ante un grupo. No importa el tamao del
grupo o el tema que expongas .Acepta la invitacin con objeto de
que vas a adquirir ms experiencia para hablar en pblico.

101

Si te invitan a platicar sobre prevencin a la farmacodependencia ante un grupo de


adolescentes, acptala. Si te invitan a hablar sobre nutricin a nios de preprimaria,
acepta. Si un club de ancianos te invita a hablar sobre motivacin, accede. Mientras
ms variados sean los pblicos y ms diversos sean los temas, es mejor, con tal de que
poseas informacin sobre ellos o suficiente tiempo para informarte.
Otra consideracin con respecto a la experiencia en hablar y el control del nerviosismo
es la formacin de un hbito, lo cual simplifica las situaciones y ayuda a controlarlas
mejor y ms fcilmente, sto ltimo evita que la tensin nerviosa se presente. El
aprendizaje de una nueva habilidad puede convertirse en un hbito. De esta forma se
liberan la atencin y la energa para dirigirlas al aprendizaje de otras habilidades. Por
ello es posible aprender muchas cosas, desde caminar hasta las ms avanzadas
operaciones filosficas y matemticas de la lgica.
Lo mismo sucede con la habilidad de hablar en pblico. Si se
aceptan las oportunidades para hacerlo adquiriremos experiencia
para controlar la situacin de comunicacin oral. La seguridad
que se adquiere al ser capaz de controlar la situacin de
enfrentarse con un pblico es el mejor remedio contra el
RELAJACIN
nerviosismo.

RELAJACIN
Para relajarse en el momento de hacer una presentacin, el orador tiene que haber
aprendido a no estar tenso en las mltiples situaciones que enfrenta en su vida
cotidiana. Para esto debe quitarse la presin e impedir que otros lo presionen. El orador
que sabe enfrentarse a la situacin de comunicacin en pblico con calma y seguridad
se conoce a s mismo. Reconoce y acepta sus limitaciones, lo mismo que sus
habilidades.
Para el orador es til:
Saber que respirar profundo antes de su presentacin le ayudar a relajarse.
Tensar sus msculos y despus aflojarlos.
Algunos oradores tienen la costumbre de sujetar con fuerza el podio y luego soltarlo
con lentitud para inducir el relajamiento en los msculos.
stas soluciones son inmediatas, por lo que es bueno tomarlas en cuenta. Sin
embargo, es difcil que ayuden mucho a quienes en general se enfrentan a mltiples
situaciones diarias con tensin y ansiedad.
Para acostumbrarse a enfrentar las situaciones de manera razonada, es bueno tener la
confianza de una persona madura con quien platicar. La autoexpresin es necesaria
para el desarrollo cognitivo, social y lingstico. El desarrollo de tales habilidades es lo
que diferencia al adulto del nio. Nuestro desarrollo psquico, del que depende la
madurez, se beneficia de la oportunidad de intercambiar las ideas prematuras,
nacientes, con una persona madura, ya sea un pariente, un maestro o un amigo que
nos aliente a expresarlas.
102

Otro factor que le ayuda a una persona a enfrentar las situaciones de forma tranquila y
calmada es su habilidad para organizarse, la cual se adquiere al aprender a optimizar
tiempo y energa. Saber organizarse facilitar cumplir con las obligaciones, de manera
que evite la tensin y el nerviosismo que se originan en el hecho de que la persona, en
apariencia, no ha contado con tiempo suficiente para desempear de forma adecuada
sus tareas.

PREPARARSE PARA LAS PREGUNTAS


Despus de la ponencia por lo general hay una sesin de preguntas y respuestas. A
veces el orador cree que no va a saber contestar los cuestionamientos, lo cual origina
preocupacin que produce nerviosismo.
Se reconocen dos tipos de preguntas:
a) Las que buscan informacin: solo requieren saber datos e informacin.
b) Las que buscan discusin: Desean descubrir las debilidades en la argumentacin
o en el razonamiento del orador.
Antes de responder una pregunta, el orador tendr que clarificarla para s mismo, para
la persona que la hizo y para el resto del pblico. Puede repetir la pregunta; preguntar a
la persona que la efectu si la ha entendido bien. A veces hay que ponerse de acuerdo
sobre la definicin del trmino. Una vez que se establece un significado comn respecto
de la pregunta entre l, la persona que la hizo y el pblico, habr que proceder a
contestarla en forma breve y precisa. Luego, si considera necesario, extender su
respuesta ofreciendo apoyos para su punto de vista.

103

CULTIVAR UNA ACTITUD RECEPTIVA


Cuando un orador est nervioso, transmite su sentimiento al pblico y lo motiva a que
sienta ansiedad por la inseguridad que l o ella demuestra.
Quiz hayas notado alguna vez que al hablar frente a un grupo, a algunas personas
parece gustarles lo que dices. Demuestran su inters al asentir con la cabeza o inclinar
el cuerpo un poco hacia delante. Por otra parte, tal vez te hayas fijado en otros
individuos muy gruones, que parece que nos quieren matar con la vista. Es lgico que
un orador se dirija a quienes le demuestran una actitud de aceptacin y evita la mirada
de aquellos que demuestran rechazo.
La persona con conocimiento de comunicacin sabe portarse como miembro de un
pblico, de forma que ayuda a aliviar la tensin del orador en vez de aumentarla. Por
medio de una expresin facial agradable y con movimientos de la cabeza apenas
perceptibles que muestran aceptacin es posible comunicar una actitud favorable y
alentadora al orador. Un buen orador tambin es un buen miembro del pblico. Sabe
hacer sentir cmodos a los otros, ya sea que est frente a un pblico comunicndole un
mensaje o, como parte de ste, recibindolo.

Malos hbitos que todo orador debe evitar:


a) Actitud nerviosa: Frente al pblico, el orador camina de un lado a otro, juega con
algn objeto o mueve los brazos y la cabeza sin sentido.
b) La inmovilidad: El orador esttico no utiliza una actividad para expresar algn
significado.
c) Exageracin de algn gesto en particular: Puede que el orador tenga un gesto
favorito que siempre utiliza, as como que excluya otros.
d) Gesticulacin exagerada: En ocasiones el orador gesticula de manera constante
en vez de utilizar tal recurso para destacar las ideas que considere importantes.
e) Relajacin exagerada: En ocasiones el orador exhibe una apariencia dbil y
aptica.
104

f) Actividad no integrada: El orador muestra diferentes grados de tensin en


distintas partes del cuerpo; por ejemplo, mover los brazos mientras mantiene una
expresin facial relajada.
g) Expresin facial fija: El orador adopta una expresin fija, como de sorpresa o de
duda, sin que se percate de ello.
h) Tics nerviosos: Es frecuente observar tics nerviosos en los msculos faciales o
en la cabeza. Estos movimientos se interpretan como signos de fuerte tensin.
i) Falta de sincronizacin entre palabras y los gestos: A veces el orador gesticula
antes o despus de haber presentado la idea que requiere el nfasis que le
confiere la gesticulacin.

Buenos hbitos que el orador debe desarrollar:

Integracin corporal: Todas las partes del cuerpo participan o responden


de manera consistente con un movimiento expresivo. Los movimientos de las
diferentes partes del cuerpo, deben ser consistentes con la expresin facial.

Significado y uso moderado: La comunicacin en pblico requiere que la


accin corporal tenga significado. Adems, la accin corporal para la
comunicacin en pblico es efectiva cuando se emplea con moderacin. Si el
orador est en constante movimiento, el pblico no apreciar cuando un
movimiento determinado lleva un significado especial.

Variedad: Para mantener la atencin del pblico el orador debe variar sus
movimientos y gestos.

Antes de decidir sobre que tema hablar, el orador requiere informacin sobre su pblico:
Cantidad: Nmero de asistentes.
Gnero: El pblico son principalmente hombres o mujeres o es un grupo
equilibrado.
Edad: Promedio de edad del pblico y cual es el rango de edades.
Nivel de Educacin: Nivel acadmico de los asistentes (elemental, media
superior, postgrado, etc.)
Cultura: El pblico es semejante en trminos tnicos, raza, nacionalidad, religin,
clase socioeconmica, etc.
Ocupacin: Tipo de actividad que realiza el pblico (enfermeras, estudiantes,
amas de casa, obreros, ingenieros, etc.)
Actividades preferidas: Polticas, profesionales, econmicas, religiosas, etc.
105

Conocimiento sobre el tema: El pblico conoce el tema de forma amplia,


moderada, mnima, etc.
Actitudes hacia el orador, el tema y el discurso: Positiva, negativa, neutral.
Adaptaciones al pblico: Argumento, lenguaje y estilo del mensaje.
Hora del discurso: Es importante saber la hora del discurso para tomar medidas
necesarias y realizar as un excelente impacto. Por ejemplo: Si el discurso est
programado despus de la comida, el pblico probablemente estar con tedio,
somnoliento (a punto de dormirse) por lo atento hay que hacer la presentacin
gil, amena y dinmica.
Entre otros.

Actividad 1: Contesta los siguientes cuestionamientos.

1. Has realizado alguna presentacin oral ante un pblico?


______________________________________________________________________
2. Cules son tus cualidades orales?
______________________________________________________________________
______________________________________________________________________
______________________________________________________________________
3. Qu haras para dominar tu nerviosismo al presentar un tema oralmente?
______________________________________________________________________
______________________________________________________________________
______________________________________________________________________
______________________________________________________________________
106

4. Qu acciones realizaras para adquirir conocimientos y dominio sobre un tema?


______________________________________________________________________
______________________________________________________________________
______________________________________________________________________
______________________________________________________________________
5. Cules fuentes de informacin utilizaras para informarte sobre determinado tema?
______________________________________________________________________
______________________________________________________________________
______________________________________________________________________
6. Si te invitaran a hablar en un evento acadmico aceptaras?
______________________________________________________________________
7. Qu factores tomaras en cuenta para organizar y presentar un tema oralmente?
______________________________________________________________________
______________________________________________________________________
______________________________________________________________________
______________________________________________________________________
______________________________________________________________________
______________________________________________________________________
______________________________________________________________________
______________________________________________________________________
______________________________________________________________________

Actividad 2: Busca en el diccionario la definicin de las siguientes palabras y


escribe su significado en las lneas.

mbito:________________________________________________________________
Habilidad:______________________________________________________________
Comunicacin:__________________________________________________________
Eficaz:_________________________________________________________________
Claridad:_______________________________________________________________
Persuasin:____________________________________________________________
Precisin:______________________________________________________________
Nerviosismo:___________________________________________________________
Pblico:________________________________________________________________
Adrenalina:_____________________________________________________________
107

Hbito:________________________________________________________________
Relajacin:_____________________________________________________________
Podio:_________________________________________________________________
Confianza:_____________________________________________________________
Autoexpresin:__________________________________________________________
Cognitivo:______________________________________________________________
Lingstico:_____________________________________________________________
Sesin:________________________________________________________________
Ponencia:______________________________________________________________
Argumentacin:_________________________________________________________
Actitud:________________________________________________________________
Oral:__________________________________________________________________
Expresin:_____________________________________________________________
Movimiento:____________________________________________________________
Gesto:_________________________________________________________________
Conferencia____________________________________________________________
Paralenguaje:___________________________________________________________
Naturalidad:____________________________________________________________
Seguridad:_____________________________________________________________
Propsito:______________________________________________________________

DEBATE

El Debate es la controversia sobre una cosa entre dos ms personas.


Debatir significa discutir, altercar, contender, disputar sobre una cosa.
El debate ofrece al pblico la oportunidad de conocer todos los aspectos en pro y en
contra de una determinada idea o asunto, mediante una continua exposicin de
argumentos, preparados con base en razonamientos lgicos y motivacionales.

108

El Debate:

Prctica persuasiva de conviccin.


Constituye la mejor influencia para la vida democrtica.
En l se aprende a ejercer la interaccin afectivo-social en un clima de respeto y
formalidad.
Puede ser usado individualmente o en grupo para asegurar la participacin de
otros.
Se considera la forma ms civilizada de llegar a tomar decisiones respecto a
hechos o sucesos en una sociedad abierta y pluralista.

El Debate es el proceso
de cuestionar y apelar la
validez de
un juicio razonado,
de una proposicin,
ya que brinda la
oportunidad de pensar
crticamente.

FUNCIN DEL DEB ATE :


1. Presentar ante un pblico a expertos o grupos de personas que con la gua de
un moderador, sostienen opiniones contrarias entre s sobre un tema conocido
por el auditorio.
2. Convencer: Es el esfuerzo persuasivo que realiza el comunicador para lograr una
accin directa sobre la mente de las personas para formar, cambiar o reforzar
sus actitudes, creencias u opiniones.
3. Persuadir: Es convencer, cambiar o influir a alguien a travs de ideas, actitudes,
palabras o comportamientos.

El beneficio social del debate y de la argumentacin es incrementar la probabilidad de


que las decisiones sobre asuntos de importancia pblica se tomen con base en
informacin adecuada, y de esta manera aumentar la calidad en la toma de decisiones.
En un debate el orador-afirmativo argumenta a favor de un cambio y el orador-refutador
argumenta en contra de ese cambio, exponiendo sus razones por las que no debe
109

haber cambio. Este proceso de argumentacin permite que los que escuchan el debate
se informen sobre las mejores razones de ambos puntos de vista. De esta manera,
obtienen la informacin necesaria para tomar decisiones adecuadas con respecto al
asunto que se debate.

En todo debate existe:


1. Un Ponente: Presenta su tesis y trata de convencer de sus ideas a todos.
2. El Oponente: Es la parte contraria, partidaria de la tesis opuesta al ponente.
3. Refutacin: Oposicin, consiste en presentar razonamientos y evidencias para
destruir su efecto. Desaprobar una idea y argumentar en su contra. El oponente
refuta al ponente con todas las armas retricas a su alcance.
El foro donde se debaten las razones a favor y en contra de un tema y como resultado
de esa discusin de puntos de vista, se obtiene una conclusin o se resuelve la
controversia, se llama contexto de argumentacin.
Al participar en un contexto de argumentacin, el ponente y el oponente deben
comenzar con un proceso de razonamiento lgico, para despus expresar sus
razonamientos en forma de argumentacin. Se razona con base en cierta informacin
antecedente que ha tenido un consecuente; este consecuente es el que ha sido
percibido o juzgado como favorable o desfavorable por las personas involucradas en el
asunto.

110

DEBATE ACADMICO

El debate acadmico, el que se realiza en universidades y escuelas


con fines de aprendizaje, ofrece grandes ventajas a quienes lo
practican como las siguientes:
Prepara para la participacin efectiva en una sociedad libre y
democrtica.
Instruye para ejercer el liderazgo.
Capacita en tcnicas de argumentacin.
Favorece la investigacin y el anlisis intensivo para problemas
contemporneos significativos.
Ayuda a la integracin de conocimientos.
Genera cuestionamientos y pensamientos propositivos.
Enfatiza la calidad de la instruccin.
Favorece la superacin en la vida estudiantil.
Conduce a una madurez social.
Despierta el espritu de lucha y fortaleza.

El debate acadmico tambin favorece el desarrollo de habilidades especficas


esenciales para el desarrollo personal, como son:
1. La habilidad de razonamiento y pensamiento crtico.
2. La habilidad de respuesta rpida y adecuada.
3. La habilidad de estructurar las ideas.
4. Las habilidades de expresin oral efectiva.
5. Las habilidades de escucha crtica.

111

Algunos temas que causan controversia son:


a)
b)
c)
d)
e)
f)
g)
h)
i)
j)
k)
l)
m)
n)
o)
p)
q)
r)
s)
t)

Realizar exmenes por computadora.


Venta de bebidas alcohlicas a menores de edad.
Cambiar el horario de verano.
Permitir el matrimonio civil a los homosexuales.
Legalizar los juegos de azar en Mxico.
Reducir la edad penal.
Legalizar el aborto.
Legalizar la pena de muerte en Mxico.
Legalizar las drogas.
Modificar los sistemas de pensiones en Mxico.
Instalar policas en las escuelas.
Los uniformes escolares deben ser obligatorios?
El matrimonio est en decadencia?
Legalizacin de vehculos extranjeros.
Los programas de televisin engendran perversin y violencia?
Penalizacin por manejar en estado de ebriedad.
Los gobiernos realmente representan y protegen a su pueblo?
Requiere la educacin en Mxico mayor inversin?
Los videojuegos causan obsesin en los nios?
Etc.

RECOMENDACIONES PAR A LA EXPOSICIN OR AL DE ARGUMENTOS


El tema debe ser cuidadosamente elegido de acuerdo con convicciones,
creencias y valores, debe ser actual y de inters para el pblico.
La propuesta del mensaje debe reflejar informacin y conocimiento de lo que
ocurre en su localidad, regin, pas o en el mundo.
Cada argumento debe apoyarse en evidencias slidas y en informacin actual y
confiable, utilizando elementos racionales y emocionales que se expresarn con
entusiasmo, naturalidad y manteniendo el contacto visual con el pblico.
La comunicacin no verbal debe proyectar seguridad, una accin corporal
espontnea y dinmica (movimientos, posicin), as como una expresin facial
sugerente y determinada. El uso del paralenguaje deber ser modulando la
entonacin para imprimir calidad de voz, y el nfasis para causar un impacto en
el pblico.

112

Los participantes deben elegir una forma de vestir formal. Cuando el debate es en
grupo, deben buscar alguna vestimenta que los identifique como grupo: todos de negro
o de blanco, o en camisa/blusa del mismo color o todos de traje, o con un elemento de
identificacin.

Actividad 3:
a) Formar equipos
b) Seleccionar un tema
c) Asignar a cada equipo una postura diferente del tema en cuestin para
que la defiendan.
d) Elegir un estudiante para moderador e iniciar el Debate.

Actividad 4: Relaciona las siguientes palabras en la Sopa de Letras


(Busca las palabras en forma vertical, horizontal y/o diagonalmente).

DEBATE
ARGUMENTACI
N
MODERADOR

PONENTE
FORO

SEGURIDAD
OPONENTE

INFLUIR
CRTICA

ENTONACIN

REFUTACIN

PERSUADIR
ACTITUD

HABILIDA
D
TEMA
PBLICO

ORADOR
APELAR

IMPACTO
INFORMACIN

M E P E R S U A P E L R C R

H E A N A R P U B L

T A T S F N L

C O E N T O N R E O E U O A

H A B T O N O A P E L A R E F U T A G K N D N L

N F A M O D E R A D E B A T

O P O R T E M P E R S U A C
U C E G

E F R A N C P A R S

D E S U R N A D

S T E M

113

R U U E

E C

O R A M N N T A T G
I

N L A T O R R E S

D N F O E O

J O L H

P M

M O D E R A D O R L A S F R

B E P E D R O L G A D S D

S L U

O N O R M

S A T

R U

N A O R T E G A U E N O U

C R

N T E

C A C T

A R G C R
O P O

C A L M A D A B T N

T U B A D A E

A C T

M P A C T A S R

N F C A R L O

S N A N C Y D

P U B D

N F O R M E O R A D U R A M A P A S

N F O R M E D E O P O N E N T E N T U P A R A S

S E G U R

R O S A R E F U T A C

C P U R E F U T A C
I

M E L D L

C A R D O T H A B

P O N O R A D O R O M E O S E G U R
D E B N U N

N O B

R O S A O R A D O

T U D O R
I

N F O R M A C

N G A P E L

O N M O D E R A R C L A R A

O N E N T O N A R

R P E R S U A M O D E R A F O R

N T E L

N F

L O C R

DISERTACIN

La disertacin consiste en reflexionar o pensar concienzudamente sobre un tema


cualquiera, con objeto de exponerlo ante un auditorio.
La conferencia o disertacin es un medio de la expresin oral, cuyo objetivo es incrementar
los conocimientos del auditorio sobre un tema determinado.
El conferenciante, al hablar de algn tema, tiene la oportunidad de ser brillante y cautivar
al pblico con su manera de hablar, valindose de recursos diversos, como libros, discos,
diapositivas, filmes, cintas magnetofnicas, etc.

114

La conferencia o disertacin no debe exceder una hora de duracin, salvo aquellos casos
en que el tema tratado sea de mucha relevancia. Ahora bien, el disertante nunca debe
olvidar las condiciones de la expresin oral.
Para que la disertacin sea un xito, el conferenciante, antes de comenzar, deber tomar
en cuenta las condiciones fsicas del local, disposicin del auditorio y el atuendo.
a) Condiciones fsicas del local
El orador debe tomar como propia la responsabilidad de verificar que:
1.- La sala de conferencias sea lo suficientemente amplia.
2.- Se prepare una mesa especial para la prensa, si es que ha sido invitada.
3.- El aire y la luz sean suficientes.
4.- Los asientos sean suficientes para el grupo de personas que han sido invitadas.
5.- Haya un micrfono y una persona que se encargue de su funcionamiento.
6.- No haya telfono para evitar distracciones.
7.- Haya agua y vasos para el conferenciante y miembros del presidium.

b) Disposicin del auditorio

115

La colocacin adecuada del pblico dentro de la sala de conferencias es de mucha


importancia, pues de esta manera se evita que el hablante haga esfuerzos para ser
escuchado por todos. El disertante debe procurar hablar de pie y no sentado como
una atencin al pblico y adems tener ms libertad de movimiento.

DISCURSO

La palabra discurso (del latn discurrere, correr en todos los sentidos) tiene dos
acepciones. Una significa la facultad del entendimiento, por medio del cual se infieren unas
cosas de otras. Por otra parte, la palabra discurso significa tambin la serie de palabras o
frases que se emplean para manifestar lo que se piensa o se siente. *
El propsito de un discurso o conferencia es:
1.- Entretener: causar agrado o complacencia en el auditorio.
2.- Informar: exposicin clara de un asunto o de una idea.
3.- Persuadir: convencer a los oyentes.

TIPOS DE DISCURSOS
116

Para determinar el propsito de un discurso conviene conocer los distintos tipos de


discursos que hay:
1.- Informativo:
Ejemplo: Tema: Dar a conocer el plan de trabajo de una planilla.
Propsito: Hacerlo as para que voten por la planilla.

2.- De entretenimiento:
Ejemplo: Tema: Lugares desconocidos de Nuevo Len donde se puede ir de
vacaciones.
Propsito: Convencer de que visitar nuestro estado es divertido.

3.- Para persuadir:


Ejemplo: Tema: El hacer las cosas de todos los das bien.
Propsito: Convencer de que el trabajo que hagamos lo debemos hacer con
agrado y esmero.

117

4.- Para formar actitudes:


Ejemplo: Tema: El liderazgo en la juventud.
Propsito: Invitar a los jvenes a ser lderes donde quiera que estn.

5.- De refutacin:
Ejemplo: Tema: No estar de acuerdo con la forma en que la televisin
manipula a los nios.
Propsito: Demostrar con argumentos el dao que la
televisin les hace a los nios.

118

* Gastn Fernndez de la Torriente, Enciclopedia prctica de la lengua, p.137

Actividad 5: Realiza las siguientes actividades.

1.- Escribe cul sera el propsito que le asignaras a los siguientes temas:
a) Tema: La violencia en el mundo.
Propsito: _____________________________________________________________
b) Tema: El hambre en las zonas marginadas de Mxico.
Propsito:______________________________________________________________
c) Tema: Los jvenes viven despreocupadamente.
Propsito:______________________________________________________________
d) Tema: Los adolescentes se apartan de la familia.

119

Propsito:______________________________________________________________
e) Tema: Comprar productos fabricados en Mxico.
Propsito:______________________________________________________________
2.- Escribe un ejemplo de cada tipo de discurso:
a) Informativo:___________________________________________
b) De entretenimiento_____________________________________
c) Para persuadir:________________________________________
d) Para formar actitudes:___________________________________
e) De refutacin: _________________________________________

3.- Menciona
conferencia:

tres

temas

sobre

los

que

te

gustara

hablar

en

una

a)____________________________________________________________________
b)____________________________________________________________________
c)____________________________________________________________________

CARACTERSTICAS DEL BUEN CONFERENCIANTE


Las caractersticas personales necesarias para lograr una comunicacin eficaz son:

Honestidad
Conocimiento
Confianza en s mismo
Habilidad verbal y no verbal
Observacin de la situacin y de las personas

Honestidad
Hace ms de dos mil aos el filsofo griego Aristteles expres una vieja verdad:
Un orador efectivo debe ser una persona efectiva. El xito de un conferenciante
-declar en su tratado de Retrica- depende no slo de su vocabulario amplio y
preciso, de su voz agradable o de sus movimientos coordinados: para tener xito, el
orador debe ser inteligente, conocer profundamente su materia y los problemas
humanos en general. Si una persona quiere lograr la aceptacin pblica de sus ideas,
debe ser respetado y considerado como individuo de calidad humana y moral.

120

Este nfasis en el carcter individual es un elemento bsico en la conferencia. Las


personas nunca escuchan slo la informacin: tambin escuchan a la persona que
presenta dicha informacin. Y ya que las palabras y los movimientos reflejan la
personalidad del expositor, el ser y la expresin del ser no se pueden separar.

Conocimiento
A travs del estudio y la observacin se podr enriquecer la experiencia y la
comprensin de los fenmenos que acontecen da a da. Lo importante, en todo caso,
es que profundice en su materia para que su informacin sea completa, precisa y
actual.

Confianza en s mismo
El conferenciante seguro de s mismo se caracteriza tanto por su actitud fsica como
por su disposicin mental. Entre otras cosas, su postura es erecta, pero no rgida sino
confortable; sus movimientos son naturales; mantiene el contacto visual con sus
receptores; su voz es vital y enrgica; y su mente alerta lo capacita para adaptar su
mensaje a la naturaleza de su auditorio y a las necesidades de la situacin.
La manifestacin de nerviosismo es una forma de inseguridad y, en cierta manera, no
slo puede obstaculizar la comunicacin, sino tambin dificultar la posibilidad de
convencer al auditorio del mrito de sus ideas.

121

Habilidad verbal y no verbal


La persuasin y el convencimiento del pblico es el objetivo que persigue un orador.
Para ello debe manejar con mucha habilidad su voz, movimientos corporales y
palabras.

Observacin de la situacin y de las personas


Una buena conferencia refleja los intereses tanto del expositor como del pblico.
Si su tema de exposicin tiene como fin cubrir un objetivo de aprendizaje, asegrese de
que su exposicin satisfaga ese requisito.

ORGANIZACIN DE UN TEMA Y SU PRESENTACIN A UN AUDITORIO


Seleccionar y delimitar un tema es lo primero que debemos hacer. Para ello hay
que tomar en cuenta lo siguiente:
1.- Basarse en experiencias personales.
2.- Que el tema sea de inters para el auditorio.
3.- Que el conferenciante sea capaz de aportar alguna idea original.
4.- Utilizar el material adecuado para la exposicin del tema.

122

Sobre qu hablar? Cmo empezar? Esto ltimo es lo ms difcil, pero se puede


hacer de la siguiente manera:
1.- Selecciona un tema (puedes elegir uno de los que mencionaste en el ejercicio
anterior).
2.- Realiza un mapa semntico o lluvia de ideas sobre las que podras hablar.
3.- Organiza tus ideas por orden de importancia, desechando las que no necesites.

Un ejemplo podra ser el siguiente:


TEM A: LA CARID AD
Mapa semntico
Madre Teresa de
Calcuta

La practican los adultos

Qu dice el evangelio
Cmo la pueden
practicar los jvenes
Definicin
PadreMaximiliano
MaximilianoKolbe
Kolbe
Padre

LA
CARIDAD

Lo que dice la iglesia


catlica
Tipos de caridad

Situaciones en que123
no se practica la caridad

Organizacin de ideas:
1.- Qu es la caridad?
2.- Los distintos tipos de caridad.
3.- Qu dice el evangelio?
4.- Qu dice la iglesia catlica?
5.- Personas que se han distinguido por practicar la caridad.
6.- Cmo la pueden practicar los jvenes y los adultos?
7.- Conclusiones personales.

Actividad 6: Organiza los temas siguientes como si fueras a exponerlos en


una conferencia. El ejemplo anterior te puede servir de ayuda.

Tema: Las diversiones.


Mapa semntico:

124

Organizacin de ideas:
______________________________________________________________________
______________________________________________________________________
______________________________________________________________________
______________________________________________________________________
______________________________________________________________________
______________________________________________________________________
______________________________________________________________________
______________________________________________________________________
______________________________________________________________________
______________________________________________________________________
______________________________________________________________________
______________________________________________________________________
______________________________________________________________________
a) Tema: La delincuencia
Mapa semntico

125

Organizacin de ideas:
______________________________________________________________________
______________________________________________________________________
______________________________________________________________________
______________________________________________________________________
______________________________________________________________________
______________________________________________________________________
______________________________________________________________________
______________________________________________________________________
______________________________________________________________________
______________________________________________________________________
______________________________________________________________________
______________________________________________________________________
______________________________________________________________________
______________________________________________________________________
______________________________________________________________________
______________________________________________________________________
b) Tema: Los vicios de los jvenes.
Mapa semntico:

126

Organizacin de ideas:
______________________________________________________________________
______________________________________________________________________
______________________________________________________________________
______________________________________________________________________
______________________________________________________________________
______________________________________________________________________
______________________________________________________________________
______________________________________________________________________
______________________________________________________________________
______________________________________________________________________
______________________________________________________________________
______________________________________________________________________
______________________________________________________________________
______________________________________________________________________
______________________________________________________________________
______________________________________________________________________
______________________________________________________________________
Existen distintas maneras de organizar las ideas de acuerdo con:
1.- El tiempo (cronolgicamente):
Ejemplo: La vida - La infancia - La adolescencia - El adulto - La vejez.
2.- El espacio:
Ejemplo: Visita a la ciudad de Mxico
Ruinas - Museos - Diversiones Teatros - Discotecas.
3.- La relacin de causa efecto (o del efecto a la causa):
Ejemplo: La crisis de la edad adulta
Caractersticas de la edad adulta - Pensamientos - Economa - Enfermedades Causas del problema - Efectos del mismo.
4.- La relacin problema-solucin (o solucin problema):
Ejemplo: El fumar
Factores que llevan a una persona a fumar - Cuando el fumar se convierte en un
problema de salud - Estrategias para dejar de fumar.
127

5.- La comparacin-contraste:
Ejemplo: La gente que vive feliz
Casos de personas que viven felices - Razn por la que viven felices Actitudes,
Comportamientos y Acciones que presentan - Compararlas con la gente que vive
entregada a los vicios.
6.- La divisin y la clasificacin:
Ejemplo: La economa de Mxico
Economa bsica - Economa de productos - Economa de consumo.
7.- El ordenamiento de lo general a lo especfico (o de lo especfico a
lo general):
Ejemplo: El amor
Definicin del amor - Tipos de amor - Casos de personas que han amado.
Una vez elegido el tema, sealados los objetivos y reunido el material para impartir la
conferencia se deber seleccionar el orden de la exposicin. El siguiente paso es
redactarla.

El orden a seguir es, por lo general, el siguiente:


1.- Un saludo al auditorio.
2.- Introduccin al tema del que se va a hablar: El conferenciante se presenta al
pblico. Esta primera parte es importante, pues es el momento oportuno para despertar
en el pblico el inters por el tema que se va a exponer.
3.- Desarrollo: Puede contener una o varias ideas importantes. En l se pueden incluir:
ancdotas, casos reales, comentarios humorsticos, ilustraciones, refranes, frases
clebres, descripciones, estadsticas, etc.
4.- Conclusiones: Es el momento culminante en el que se redondean y adquieren
pleno sentido las ideas que se expusieron. El conferenciante puede entonces:
Recordar el objetivo de la conferencia discurso.
Citar lo que otra persona dijo respecto al tema que se expuso.
Invitar al pblico a reconsiderar su actitud respecto al tema que se trat.
Hacer una recomendacin especfica sobre el tema expuesto, etc.

128

Una vez que se ha redactado la conferencia, conviene revisarla y pensar sobre la


manera en que se expondr al pblico, as como en el material de apoyo que se va a
utilizar.

HOJA DE EVALU ACIN


Tema de debate: ________________________________________________________
Nombre del participante: __________________________________________________
Usa esta forma para evaluar a tus compaeros en su presentacin. D una calificacin
del 1 al 5 al participante.
Considerando: 5 = Excelente, 4 = Muy Bien, 3 = Bien, 2 = Regular y 1 = Mala
__________

A. Documentacin del tema: Sus opiniones estn fundamentadas?


Cit fuentes, bibliografa, etc.?

__________

B. Desarrollo de ideas, juicios:


congruencia en las ideas?

__________

C. Desenvolvimiento del orador: Se ve seguro? Muestra fluidez al


hablar, buen tono de voz, buena diccin? Tiene movimientos
129

Hay

secuencia,

claridad

corporales de apoyo?
__________

D. Grado de conviccin: Los argumentos son convincentes? El


orador logra persuadir?

__________

E. Organizacin y uso del tiempo: Son adecuados?

Comentarios:___________________________________________________________
______________________________________________________________________
______________________________________________________________________
______________________________________________________________________
______________________________________________________________________
______________________________________________________________________
______________________________________________________________________
______________________________________________________________________
______________________________________________________________________

Actividad 5: Elabora una conferencia para exponerla ante tus compaeros. Recuerda:
a)
b)
c)
d)

Elige el tema de la conferencia.


Sigue los pasos indicados para su preparacin.
Ensyala sirvindote del material de apoyo.
Exponla ante tus compaeros en la fecha que te indique el maestro.

130

AUTOEVALUACIN
Instrucciones: Lee cada reactivo y selecciona la respuesta correcta.
Lee el siguiente texto e identifica la caracterstica de la Expresin Oral que est
contenida en l.
1.- En el examen final de la materia el maestro te solicita que selecciones un tema y
realices una exposicin oral en el auditorio. Cmo manifestaras el Autodominio en
tu presentacin si un participante expresara verbalmente y ante todos los asistentes
que no est de acuerdo con tu punto de vista?
A) "Comprendo tu preocupacin y punto de vista. Permteme explicarte mis razones
y comprenders por qu lo dije".
B) "Tu eres de primer semestre y no sabes mucho sobre este asunto."
C) Utilizar todos mis argumentos para convencerte de que tengo la razn.
D) No dispongo de tiempo para debatir contigo el tema.

131

Identifica en el siguiente texto el estado de nimo del alumno y selecciona la opcin que
ms le ayudara a superar exitosamente la situacin:
2.- Un estudiante participante en el Concurso Estatal de Oratoria del CECyTE N.L.
antes de que le toque su turno se pone muy nervioso, le estn sudando las manos, le
tiemblan las piernas y se siente mareado. Qu debe hacer?
A)
B)
C)
D)

Tener confianza en s mismo, relajarse, y adoptar una actitud seria.


Pedir un vaso de agua y un tranquilizante.
Retirarse, tomar aire fresco y no participar.
Repasar rpidamente sus apuntes y pedirle a un compaero que lo escuche.

Identifica la opcin que seala el procedimiento correcto que debe seguir un estudiante
en el siguiente caso.
3.- El Director de un CECyTE comision a una alumna del 2 semestre para que
expusiera ante nios de primaria el tema Por qu es importante el CECyTE? La
alumna requiere informacin del pblico que asistir.
Cules son los aspectos ms relevantes que debe conocer?
A) Buenos hbitos de los asistentes, cantidad y domicilio.
B) Cantidad, hora del discurso, lugar, duracin y expectativas de los nios.
C) Hora del discurso y aprovechamiento escolar.
D) Estadstica por gnero, turno, por ciento de reprobacin y micrfono.
4.- Identifica la actitud que mostr un conferenciante en el siguiente caso:
El conferenciante abord el tema Los Valores en la Escuela y los padres de familia
mostraron gran inters que se evidenci cuando la mayora asenta con la cabeza.
A) Incomodidad
B) De aburrimiento
C) Receptiva
D) Crtica
5.- Conocemos importantes elementos de la prehistoria gracias a las pinturas rupestres de
Altamira, Espaa, esto es a la comunicacin iconogrfica, si tu quisieras dejar un mensaje
a las futuras generaciones utilizando la comunicacin oral cules recursos utilizaras?
Identifcalos.
A) CD, Video, Headphone.
B) Pintura, escultura, libro.
C) Grabaciones de voz en diversos sistemas.
D) Cine, pintura, disket.

132

6. Con motivo de la iniciativa de ley que autorizara la posesin de cierta cantidad de


drogas ilcitas para consumo personal, una estacin televisora convoc a un Diputado,
un padre de familia, un mdico y un socilogo para que expusieran sus puntos de vista
Qu tipo de interlocucin crees que utilizaron?
A)
B)
C)
D)

Disertacin
Debate
Discurso
Conferencia

7. Cinco candidatos a la presidencia se reunieron para debatir sobre sus propuestas de


gobierno. El evento fue en el programa de televisin conducido por Joaqun Driga,
quien los present por orden alfabtico de su apellido. Quin fue el tercer ponente?
A) Patricia Mercado
B) Roberto Madrazo
C) Andrs Manuel Lpez Obrador
D) Valentin Campa
Lee el siguiente texto y contesta las preguntas 8, 9, 10 Y 11.
En un plantel escolar varios alumnos se enfermaron del estmago y se sospecha
que ingirieron algn alimento que los da, el cual compraron en la Tienda de la
esquina. El maestro de LEXO organiz un debate sobre el tema: Los alimentos
chatarra daan la salud de los jvenes en el que participaron un mdico, una
nutriloga, un alumno del plantel, un ingeniero y el dueo de la tienda.
Cada uno de los participantes defendi su punto de vista ya sea a favor o en
contra del tema.
El mdico y la nutriloga argumentaban la importancia de comer frutas, verduras
y carne para mantenerse sano, mientras que el estudiante de tu plantel y el ingeniero
refutaban que era ms rico y barato comer fritos, dulces y refrescos. El dueo de la
tienda comentaba que el venda lo que ms compraban los estudiantes que por
supuesto son lonches, frituras, refrescos y dulces.
En este evento tambin particip un estudiante de 6 semestre de tu plantel que se
encarg de abrir la sesin a tiempo, presentar a los participantes, anunciar el asunto a
tratar y el orden a seguir, conceder el uso de la palabra a cada participante y mantener
el debate dentro de las reglas establecidas.
Cabe mencionar que la estudiante de nutricin en todo el Debate se mostr atenta a lo
que decan los participantes y el pblico, as como a contestar atinadamente y sin
rodeos las preguntas que le haca el pblico.
8.- Cul es el papel que desempe el estudiante de 6 semestre en el Debate?
A) Moderador
B) Refutador
C) Oponente
133

D) Ponente
9.- Qu rol desempearon en el Debate la nutriloga y el mdico?
A) Moderador
B) Refutador
C) Oponente
D) Ponente

10.- Qu destreza mostr la estudiante de nutricin en el Debate?


A) Saber escuchar
B) Conocer a sus adversarios
C) Mantener el buen humor
D) Tener autoestima

11.- De acuerdo al texto que leste cul crees que sea la funcin del debate?
A) Convencer, advertir, y entretener
B) Presentar opiniones, convencer y persuadir
C) Analizar un tema, persuadir y comprender
D) Argumentar, discutir y analizar
Lee el siguiente texto y contesta las preguntas 12, 13, 14 y 15.
En el auditorio del municipio donde vives se llev a cabo una conferencia con el tema
El Sueo Americano.
Para su presentacin el orador utiliz diapositivas, msica y material impreso (posters y
folletos).
Una parte del discurso fue la siguiente:
Muchos mexicanos se han ido de nuestra Patria buscando mejores condiciones de vida,
teniendo que enfrentar tristeza, amargura, coraje e impotencia al dejar a su gente.
Los indocumentados mexicanos que se fueron siguiendo las luces de un sueo, que han
renunciado a sus costumbres y familias, y que abandonaron sus tierras y sus casas por un
futuro prometedor en los Estados Unidos
Nuestros peores enemigos, no son los gringos que explotan a nuestra gente en sus
campos y comercios, y que mandan a los nuestros a morir por ellos al frente de sus ejrcitos.
No! Nuestros peores enemigos somos nosotros mismos. Nosotros... Nosotros que ignoramos a
nuestros hermanos de raza y que nos enorgullecemos por tener buenas relaciones con los
Estados Unidos...

134

Vaya gobierno el nuestro que antepone sus intereses polticos a las necesidades de los
indocumentados, y que toma sus problemas como pretexto para justificar su falta de trabajo. Lo
que pasa seores, es que tanto nos interesa quedar bien con el vecino, que pisoteamos la
dignidad de muchos mexicanos que s sudan para ganarse el pan de cada da

12.- Identifica el tiempo ideal que el conferenciante utiliz para mantener interesado al
pblico.
A) Tres horas
B) Dos horas
C) Una hora
D) Dos horas y media
13.- Identifica el motivo fundamental por la que el conferenciante utiliz material
impreso en su conferencia.
A) Para impresionar al pblico.
B) Para que el pblico se estremeciera con las imgenes y analizara las ventajas y
desventajas antes de decidir irse a E.U.
C) Para cautivar al pblico haciendo su presentacin ms brillante.
D) Para que luciera ms atractivo el auditorio.
14.- Cmo se le llama la presentacin que expuso el conferenciante ante el auditorio
con el objetivo de que el pblico incrementara sus conocimientos sobre el tema y en la
que hizo reflexionar sobre El Sueo Americano?
A) Disertacin
B) Debate
C) Discurso
D) Simposium
15.- Para que la conferencia sea un xito, cules fueron las condiciones fsicas del
auditorio que el conferenciante debi tomar en cuenta?
A) Aire y luz suficientes, micrfono en buen funcionamiento y suficientes asientos para
el pblico invitado.
B) Micrfono en buen funcionamiento, mesa para la prensa y una silla para el
conferenciante.
C) Suficientes asientos para el pblico invitado y un telfono disponible para el
conferenciante.
D) Aire y luz suficientes, agua y refrescos para el conferenciante.

135

16.- En el plantel en que estudias hubo una ponencia sobre Competencias Laborales
y el expositor expres que el 90 % de las empresas regionales solamente est
contratando jvenes con estudios de bachillerato terminado. Cul crees que haya sido
la intencin del participante al comunicar esto?
A) Informar
B) Entretener
C) Regaar
D) Persuadir

Lee el siguiente texto y contesta las preguntas 17, 18 y 19.


Para la ceremonia de graduacin invitan a un estudiante de 4 semestre a pronunciar
el discurso de despedida. Durante la presentacin el alumno mantuvo el contacto visual
con sus receptores, su postura fue confortable, su voz vital y enrgica y sus
movimientos naturales.
El Discurso fue el siguiente:
Jvenes graduandos busquen siempre el equilibrio en la vida. No disminuyan su
propio valor comparndose con otros. Todos somos diferentes y cada uno es especial. Slo
ustedes estn en condiciones de elegir lo que es mejor para ustedes.
Apguense a sus ideales. No dejen que la vida se les escurra entre los dedos por vivir
en el pasado o para el futuro. No abandonen cuando todava son capaces de un esfuerzo ms.
Nada termina hasta el momento en que uno deja de intentar.
No teman enfrentar riesgos, es corriendo riesgos que aprendemos a ser valientes. No
corran tanto por la vida que lleguen a olvidar no slo donde han estado sino tambin a dnde
van. No teman aprender, el conocimiento es liviano, es un tesoro que se lleva fcilmente. No
usen imprudentemente el tiempo o las palabras, no se pueden recuperar. La vida no es una
carrera, sino un viaje que debe ser disfrutado a cada paso

17.- Qu tipo de comunicacin utiliz


emotividad, mmica y gesticulacin?

el alumno si emple entonacin de voz,

A) Comunicacin Verbal
B) Comunicacin Social
C) Comunicacin no Verbal
D) Comunicacin Efectiva
18.- Identifica el tipo de discurso que pronunci el alumno.
A) Informativo
B) De entretenimiento
C) De refutacin
136

D) De formacin de actitudes
19.- Identifica la caracterstica del Buen Conferenciante que mostr el estudiante en la
presentacin del discurso.
A) Observacin de la situacin
B) Confianza en s mismo
C) Honestidad
D) Conocimiento
Lee el siguiente texto y contesta la pregunta.
Para iniciar una conferencia con el tema La familia est en decadencia? El
conferenciante relata una historia y ofrece algunos antecedentes. Mediante una
declaracin inesperada y sorprendente plantea las siguientes preguntas Se han
imaginado vivir sin la familia? Han pensado que sera de ustedes sin nadie que los
apoye? Podran imaginarse una sociedad sin familias? Qu hacen los padres de
jvenes que no respetan a sus semejantes y roban, se drogan, no estudian y provocan
problemas a la sociedad?
20.- Por qu crees que el conferenciante realiz estas preguntas?
A) Para captar la atencin del pblico
B) Para demostrar su conocimiento del tema
C) Para entretener al pblico
D) Para conocer cuanto sabe el pblico del tema

BIBLIOGR AFA
Ramos Alas, Mara Teresa
Espaol 3
Oxford University
Editorial Press Harla

Fonseca Yerena, Socorro


Comunicacin Oral y Escrita
Fundamentos y Prctica Estratgica
Editorial Prentice Hall

Barragn Camarena, Jorge


Etimologas Grecolatinas
3. edicin
Publicaciones Cultural
Mxico 2004

McEntee, Eileen
Comunicacin Oral
2. edicin
Editorial Mc Graw Hill

Diccionario Consultor Espasa


Casa Editorial Calpe S.A.
1998

Verderber F., Rudolph


Comunicacin Oral y Efectiva
11. edicin
137

Editorial Prentice Hall


Del Ro, Mara Asuncin
Taller de Redaccin I
2. edicin
Editorial Mc Graw Hill

Pineda R., Mara Ignacia


Lectura y Redaccin con Anlisis Literario
Editorial Prentice Hall

De la Torre, Francisco J.
Taller de Lectura y Redaccin II
3. edicin
Editorial Mc Graw Hill

Rangel Hinojosa, Mnica


Comunicacin Oral
ANUIES, reimpr. 1995
Editorial Trillas

Zarzar Charur, Carlos


Taller de Lectura y Redaccin I
Editorial Publicaciones Cultural
Mxico 2004

Martnez Garca, Esperanza


Taller de Expresin Oral y Escrita 2
Mxico
Editorial Trillas
1998

Len Meja, Alma B.


Estrategias para el Desarrollo de la
Comunicacin Profesional
Editorial LIMUSA, Noriega Editores.

Oseguera, Eva Lidia


Taller de Lectura y Redaccin I
2. Edicin
Mxico, 2000
Publicaciones Cultural

NOTAS

138

139

S-ar putea să vă placă și